ABFM CKSA 22-23

Lakukan tugas rumah & ujian kamu dengan baik sekarang menggunakan Quizwiz!

A 57-year-old male recently diagnosed with acute lymphoblastic leukemia presents to the emergency department with intractable nausea, vomiting, and myalgias. His first chemotherapy infusion was administered earlier in the day. Which one of the following electrolyte disturbances would be consistent with tumor lysis syndrome? A. Hypocalcemia B. Hypokalemia C. Hyponatremia D. Hypophosphatemia E. Hypouricemia

ANSWER: A Hypocalcemia can happen in tumor lysis syndrome, which may occur as a complication of chemotherapy in hematologic malignancies, such as acute leukemia or lymphomas. Calcium levels drop as serum binds with free phosphorus released from the cells and a depletion of calcium in the bloodstream. Acute cell lysis releases phosphorus, potassium, calcium, and uric acid into the bloodstream resulting in hyperphosphatemia, hyperkalemia, and hyperuricemia. Sodium levels are not typically affected.

A 70-year-old male presents with lower extremity pain. Increased pain with which one of the following would be most consistent with lumbar spinal stenosis? A. Lumbar spine extension B. Lumbar spine flexion C. Internal hip rotation D. Pressure against the lateral hip and trochanter

ANSWER: A Intermittent neurogenic claudication, also known as pseudoclaudication, is a common clinical presentation of central spinal stenosis. Spinal extension decreases the cross-sectional area of the spinal canal and leads to compression and ischemia of the spinal cord as well as narrowing of the intervertebral foramina spaces. This pathophysiology is most generally manifested as diffuse low back pain, diffuse pain or radiculopathy in the buttocks and/or lower extremities, and leg weakness that worsens with upright walking and improves with bending forward or sitting. This has been called the "shopping cart sign" where a person is most comfortable walking if they can lean forward on a shopping cart or other object. Pain with internal hip rotation is characteristic of hip arthritis and is often felt in the groin. Pain in the lateral hip is more typical of trochanteric bursitis.

A 24-year-old female presents with a 2-day history of mild to moderate pelvic pain. She has had two male sex partners in the last 6 months. She currently has a levonorgestrel (Mirena) IUD in place. A physical examination reveals a temperature of 36.4°C (97.5°F) and moderate cervical motion and uterine tenderness. A urine hCG test and a urinalysis are negative. Vaginal microscopy shows only WBCs. The initiation of antibiotics for the treatment of pelvic inflammatory disease in this patient A. is appropriate at this time B. requires an elevated temperature, WBC count, or C-reactive protein level C. should be based on the results of gonorrhea and Chlamydia testing D. should be based on the results of pelvic ultrasonography E. should be done only after removal of the IUD

ANSWER: A Pelvic inflammatory disease (PID) is a clinical diagnosis, and treatment should be administered at the time of diagnosis and not delayed until the results of the nucleic acid amplification testing (NAAT) for gonorrhea and Chlamydia are returned. Mild to moderate disease can be treated in an outpatient setting with a single dose of an IM cephalosporin (i.e. ceftriaxone 500 mg for individuals <150 kg or 1 g for individuals ≥150 kg) along with oral doxycycline (100 mg BID) and metronidazole (500 mg BID) for 14 days, if the patient is not allergic to those medications. The clinical diagnosis is based on an at-risk woman presenting with lower abdominal or pelvic pain, accompanied by cervical motion, uterine, or adnexal tenderness that can range from mild to severe. There is often a mucopurulent discharge or WBCs on saline microscopy. Acute phase indicators such as fever, leukocytosis, or an elevated C-reactive protein level may be helpful if the diagnosis is in question but are neither sensitive nor specific. A positive NAAT is not required for diagnosis and treatment because an upper tract infection may be present, or the causative agent may not be gonorrhea or Chlamydia. PID should be considered a polymicrobial infection. Pelvic ultrasonography may be used if there is a concern about other pathology such as a tubo-ovarian abscess. An IUD can be left in place as long as the patient responds to initial treatment as expected.

A 49-year-old female presents to your office in northern California with a 6-week history of increasing cough, wheezing, fever, and shortness of breath. She has type 2 diabetes and persistent asthma treated with high-dose budesonide/formoterol (Symbicort), 160/4.5 µg. Two weeks ago she went to the urgent care clinic and was treated with azithromycin and a 5-day course of prednisone. Her symptoms initially improved but have worsened in the past 3 days. She has not traveled recently. On examination today she has a temperature of 37.7°C (99.9°F), a respiratory rate of 22/min, and an oxygen saturation of 92% on room air. A pulmonary examination is significant for diffuse expiratory wheezing. A chest radiograph shows some interstitial thickening indicative of bronchiectasis. A CBC is significant for eosinophilia with an eosinophil count of 960/mm3 (N 30-350). Testing for which one of the following organisms is most likely to reveal a contributing factor to her illness? A. Aspergillus B. Coccidioides immitis C. Histoplasma D. Mycobacterium tuberculosis E. Pneumocystis jirovecii

ANSWER: A Pulmonary aspergillosis comprises a spectrum of clinical disease, from invasive, often cavity-forming disease in critically ill and profoundly immunosuppressed patients to allergic bronchopulmonary aspergillosis. This case of poorly controlled asthma associated with eosinophilia and bronchiectasis is typical of allergic bronchopulmonary aspergillosis. Aspergillus IgE titers are recommended as initial testing in patients with suspected allergic bronchopulmonary aspergillosis. Antifungal treatment can improve outcomes in these cases. Coccidioides immitis and Histoplasma typically cause more systemic symptoms such as muscle and joint pain, rather than wheezing. Coccidioides immitis is present in the desert regions of the U.S. Southwest and Histoplasma is endemic to the Mississippi and Ohio River valleys of the Midwest and the South. Mycobacterium tuberculosis is less common in the United States, and this patient's symptoms and findings are not typical of active tuberculosis. Pneumocystis jirovecii causes pneumonia but is rare in patients who are immunocompetent.

Which one of the following clinical features is a component of the STOP-Bang questionnaire used as a screening tool for obstructive sleep apnea? A. Blood pressure B. Pulse rate C. Resting oxygen saturation D. Smoking status E. Waist circumference

ANSWER: A The STOP-Bang questionnaire is a screening tool to help identify patients with obstructive sleep apnea (OSA). In the questionnaire S = snoring, T = tiredness, O = observed apnea, P = high blood pressure, B = BMI >35 kg/m2, A = age >50, N = neck circumference >40 cm, and G = male gender. For each question, answering "yes" scores 1, answering "no" scores 0, and the total score can range from 0 to 8, with a higher score indicating a higher probability of OSA. A score of at least 3 has >90% sensitivity for moderate to severe OSA.

Which one of the following is the fundamental counseling strategy used in the Stages of Change Model? A. Quickly establishing rapport with a patient to improve compliance with recommendations for change B. Assessing the patient's motivation for change and determining where they are in the process C. Focusing on a specific aspect of a problem and offering strategies for coping D. Providing education regarding the behavior in which a change is recommended E. Providing direct advice regarding steps for making a lifestyle change

ANSWER: B The Stages of Change Model assesses the patient's motivation for change and determines which stage of the change process the patient is in. The stages include precontemplation, contemplation, preparation, action, and maintenance. Understanding this helps guide various counseling strategies that family physicians can utilize for each individual patient.

A 55-year-old male is hospitalized because of altered mental status. His group home caregiver reports a 1-day history of the patient being confused and unable to perform his activities of daily living. He has a history of hypertension, COPD, and bipolar disorder, and his medications include losartan (Cozaar), inhaled tiotropium (Spiriva), and valproic acid. On examination the patient's vital signs are normal and he is oriented to person, but not to time or place. The remainder of the physical examination is within normal limits. Laboratory findings, including liver enzymes and renal function tests, are normal except for an elevated ammonia level. Ultrasonography of the abdomen does not show any liver abnormalities. Lactulose therapy is started. Which one of the following is the most likely cause of this patient's elevated ammonia level? A. Valproic acid B. Occult upper gastrointestinal bleeding C. Portal vein thrombosis D. Gilbert syndrome

ANSWER : A Patients treated with valproic acid (VPA) are at risk for developing hyperammonemia. While this is often asymptomatic, encephalopathy can occur. In addition to discontinuing valproic acid, the encephalopathy should be managed with ammonia-lowering drugs such as lactulose. Occult gastrointestinal bleeding is not associated with hyperammonemia. It should be suspected in patients with an elevated BUN level in the absence of underlying renal disease or volume depletion. Patients with portal vein thrombosis present with abdominal pain and other symptoms of an underlying predisposing disease such as cirrhosis. Gilbert syndrome causes an asymptomatic elevation of bilirubin in the absence of underlying hepatic disease

A patient's office spirometry results demonstrate an obstructive pattern. This would be seen with which one of the following? A. Asbestosis exposure B. Cystic fibrosis C. Idiopathic pulmonary fibrosis D. Nitrofurantoin exposure E. Sarcoidosis

ANSWER: B Office spirometry can be very helpful in narrowing the differential diagnosis of dyspnea. Of the options listed, only cystic fibrosis can cause an obstructive pattern. Other causes of an obstructive pattern include asthma, COPD, α1-antitrypsin deficiency, and bronchiectasis, among others. Common diseases or conditions causing restrictive patterns include adverse reactions to nitrofurantoin, methotrexate, and amiodarone. Chest wall conditions such as kyphosis, scoliosis, and morbid obesity can also cause restrictive patterns. Interstitial lung disease, including idiopathic pulmonary fibrosis, sarcoidosis, and asbestosis, also causes a restrictive pattern.

A 26-year-old G2P1001 at 30 weeks gestation was recently diagnosed with gestational diabetes and is ready to start testing her blood glucose at home. Which one of the following is the recommended goal for fasting blood glucose in this patient? A. <75 mg/dL B. <95 mg/dL C. <120 mg/dL D. <150 mg/dL

ANSWER: B The goal fasting blood glucose level in patients with gestational diabetes is <95 mg/dL. A fasting glucose level <80 mg/dL is associated with increased maternal and fetal complications. The goal 2-hour postprandial glucose level is <120 mg/dL and the goal 1-hour postprandial glucose level is <140 mg/dL.

The inability to use and make sense of numbers is a common problem encountered in physician-patient communication and can make it difficult to achieve shared decision-making. Which one of the following methods has been shown to be a helpful strategy when discussing numbers? A. Using relative risk instead of absolute risk B. Using icon arrays (pictographs) to show ratios C. Using percentages instead of frequencies D. Framing outcomes in either positive or negative terms, but not both

ANSWER: B The use of simple graphical representations and other visual aids can greatly enhance a patient's comprehension of numbers. One useful tool is expressing ratios as an icon array in which a shape is repeated a specific number of times to represent the denominator and some of the shapes are shaded to represent the numerator. Other techniques include using absolute risk instead of relative risk, using frequencies instead of percentages, and framing outcomes in both positive and negative terms.

According to the Ottawa knee rule, a radiograph would be indicated for a patient presenting with an acute knee injury if the examination reveals tenderness to palpation over the A. fibular head B. lateral joint line C. medial joint line D. tibial tubercle E. upper medial aspect of the tibia

ANSWER : A The Ottawa knee rule is a validated clinical practice tool that decreases unnecessary radiographs in patients with an acutely injured knee. Because of the low prevalence of clinically significant fractures in patients with acute knee injuries, radiographs should be limited to patients who meet specific evidence-based criteria. Criteria for imaging according to the Ottawa knee rule include any of the following: - age >55 - isolated tenderness of the patella - tenderness of the fibular head - inability to flex the knee to 90° - inability to bear weight for four steps both immediately after the injury and at the time of the examination. Validation studies for the Ottawa knee rule have found that the application of this rule is able to detect all fractures in the sample population (sensitivity 98%-100%), and about 50% of the patients who were "ruled in" by the Ottawa knee rule had a fracture (specificity 49%). The fracture rate in the validation studies ranged from 5% to 11%. Pain over the lateral or medial joint line is more likely to result from meniscal derangements or a sprain or rupture of a collateral ligament. Pain over the tibial tubercle is more typical of Osgood-Schlatter disease (tibial apophysitis). Pain over the upper medial aspect of the tibia suggests pes anserine bursitis.

You are called by the parents of a 6-year-old male because he has a 2-week history of awakening at night with severe back pain. You request an immediate evaluation in your office. A likely cause of this pain is A. juvenile idiopathic arthritis B. lumbar sprain C. compression fracture D. diskitis E. scoliosis

ANSWER: D Back pain that regularly occurs at night and awakens a child is usually associated with tumors or infections, such as osteomyelitis, diskitis, osteoid osteoma, osteoblastoma, and spinal cord tumors. Other possible symptoms associated with these serious causes of nighttime back pain include fever, malaise, and weight loss. Back pain that occurs at night is an indication for an expeditious and thorough medical evaluation. Juvenile idiopathic arthritis is characterized by prominent systemic symptoms such as fever and rash with arthralgias. By definition it is a chronic condition, persisting >6 weeks. Back pain alone would be an unlikely presentation. A lumbar sprain and scoliosis do not typically cause severe nighttime pain. A compression fracture would be rare in this age group without significant trauma or an underlying risk factor.

A 27-year-old soccer player presents with anterior hip pain along with a clicking sensation in the hip when he runs or attempts lateral movements. The flexion, adduction, and internal rotation (FADIR) test and the flexion, abduction, and external rotation (FABER) test both elicit pain. You suspect a labral tear. What is the most accurate imaging test for the suspected diagnosis? A. anteroposterior and frog-leg lateral radiographs of the hip B. CT of the hip C. a bone scan D. MRI E. a standing radiograph of the hip

ANSWER: D Patients with labral tears usually present with anterior hip pain and may have catching, popping, or clicking sounds associated with activities such as gymnastics, soccer, dancing, basketball, or hockey. On physical examination the flexion, adduction, and internal rotation (FADIR) test and the flexion, abduction, and external rotation (FABER) test will elicit pain. Although initial imaging may include radiographs of the hip, MRI is often needed for diagnosis. MR arthrography with gadolinium injection into the hip joint has been the standard to diagnose labral tears, but newer MR technology (3-tesla MRI with specialized protocols) allow for imaging of the joint without the need for intra-articular contrast. Neither CT of the hip nor a bone scan are recommended imaging modalities for suspected labral tears.

A 52-year-old male sees you for routine follow-up of diabetes mellitus. His hemoglobin A1c is 7.6%. He also notes that he has been having increasing pain in his right shoulder over the past few months but he cannot recall any specific injury. The pain is a dull, poorly localized ache that radiates into the biceps and is aggravated when he reaches overhead. On examination you note decreased active and passive range of motion in forward flexion, internal and external rotation, and abduction. There is normal strength with resisted activation of the rotator cuff muscles. Impingement testing is negative but is limited by his decreased range of motion. Which one of the following is the most likely diagnosis? A. Adhesive capsulitis B. Osteoarthritis of the shoulder C. Superior labral anterior to posterior (SLAP) lesion D. Infraspinatus tendinopathy E. Supraspinatus tendinopathy

ANSWER: A Adhesive capsulitis is characterized by worsening shoulder pain that is hard to localize. The underlying pathology is contraction of the glenohumeral capsule resulting in decreased active and passive range of motion. It is an idiopathic condition but has an increased prevalence in patients with diabetes mellitus and hypothyroidism. Adhesive capsulitis is often self-limited but can persist for years in some patients. Nonsurgical treatment options include physical therapy, oral or intra-articular corticosteroids, acupuncture, and hydrodilatation. Osteoarthritis is more common among older patients and typically develops more chronically than the subacute presentation of adhesive capsulitis. There is often a history of trauma, previous surgery, or repetitive work. Superior labrum anterior to posterior (SLAP) lesions can be acute (e.g., sustained during a fall on an outstretched hand) or chronic (e.g., in throwing athletes or other high-risk professions). Passive range of motion may be sustained. Infraspinatus tendinopathy may show pain with resisted external rotation. It may also arise due to acute injury or chronic repetitive overuse. Supraspinatus tendinopathy may present with normal strength but positive impingement testing as well as a positive Jobe test or empty can test.

A 60-year-old male presents with a 4-week history of weight loss, intermittent fever, and worsening fatigue. On examination you note multiple sub-centimeter nontender cervical lymph nodes and a single 2-cm axillary lymph node without other abnormal findings. Which one of the following is the best next step in management? A. CT of the chest, pelvis, and abdomen with and without contrast B. PET-CT scan C. A bone scan D. Bone marrow aspiration and biopsy E. Open lymph node biopsy

ANSWER: A This patient's symptoms of fever, fatigue, and weight loss are consistent with an oncologic cause for his lymphadenopathy. The initial evaluation for lymphoma or metastatic disease includes hematologic studies and CT imaging to assess for other lymphadenopathy or a primary tumor. PET-CT scan, a bone scan, bone marrow aspiration, and open lymph node biopsy are all possible later parts of the diagnostic pathway, particularly if lymphoma or metastatic cancer remain high on the differential diagnosis.

You are initiating pharmacologic therapy for a 75-year-old patient with depression. Which one of the following would be most appropriate for this patient? A. Amitriptyline B. Escitalopram (Lexapro) C. Imipramine (Tofranil) D. Paroxetine (Paxil)

ANSWER: B Escitalopram is a preferred antidepressant for older patients. Amitriptyline and imipramine are highly anticholinergic and sedating, and can cause orthostatic hypotension according to the Beers Criteria. They have an "avoid" recommendation. Paroxetine has a relatively high antihistamine effect and should generally be avoided in older patients due to a higher likelihood of adverse effects.

Which one of the following comorbidities would falsely lower the hemoglobin A1c level in a patient with type 2 diabetes mellitus? A. Vitamin B12 deficiency B. Iron deficiency anemia C. Hemolytic anemia D. Chronic kidney disease E. A history of splenectomy

ANSWER: C Erythrocyte (RBC) lifespan can be altered due to changes in RBC turnover, which subsequently can alter the hemoglobin A1c value. When the mean erythrocyte lifespan is reduced by conditions such as hemolytic anemia, hemoglobin A1c is decreased because of lower RBC exposure time for glycation. In contrast, when the mean erythrocyte lifespan is increased by a condition such as asplenia, hemoglobin A1c increases because of increased RBC exposure time for glycation. Conditions that decrease erythropoiesis, such as iron deficiency anemia, also increase the mean age of the RBC, thereby increasing hemoglobin A1c. Severe chronic kidney disease may increase RBC glycation through lipid peroxidase of hemoglobin and by extending the erythrocyte lifespan due to decreased erythropoietin levels, causing a false elevation of hemoglobin A1c. Vitamin B12 deficiency also decreases erythropoiesis and leads to falsely elevated hemoglobin A1c.

A 35-year-old male first presented to your office 4 months ago with a persistent chronic cough. He has no significant past medical history and has never smoked or vaped. Over the past few months he has been evaluated for GERD, asthma, eosinophilic bronchitis, and upper airway cough syndrome without symptomatic relief or diagnosis. Which one of the following is recommended for chronic refractory cough in this otherwise healthy male? A. Cyclobenzaprine B. Duloxetine (Cymbalta) C. Gabapentin (Neurontin) D. Lorazepam (Ativan) E. Propranolol

ANSWER: C In randomized, controlled trials, gabapentin has demonstrated benefit for treating unexplained chronic cough. Unexplained chronic cough is defined as a cough that persists longer than 8 weeks, despite investigation and therapeutic trials. Testing for bronchial hyperresponsiveness and eosinophilic bronchitis, or a therapeutic corticosteroid trial should be considered. Cyclobenzaprine, duloxetine, lorazepam, and propranolol have not proven to be beneficial in reducing or eliminating chronic cough.

The U.S. Preventive Services Task Force recommends which one of the following for prevention of falls in community-dwelling adults ≥65 years of age who are at increased risk for falls? A. Empirical vitamin D supplementation B. Psychological evaluation and treatment programs C. In-home environmental evaluation and modification D. Regular participation in a structured exercise program

ANSWER: D The USPSTF recommends exercise interventions to prevent falls in community-dwelling adults ≥65 years of age who are at increased risk for falls (B recommendation). Falls occur at least once annually in approximately 30% of community-dwelling adults 65 years or older, and the USPSTF recommendation is based on studies that demonstrated improved fall-related outcomes for patients who participated in exercise programs. Most evidence included group exercise, but supervised individual interventions also appeared to be of benefit. Gait, balance, and functional training; resistance training; flexibility work; and endurance training all appeared to be effective for reducing falls. Other interventions associated with a reduction in falls or fall-related fractures include an annual fall risk assessment, assistive devices such as grab bars or walkers, and environmental assessment and modifications such as the removal of trip hazards from the home. However, the evidence for in-home assessment of environmental hazards is less robust. There is no evidence for psychological evaluation in the prevention of falls. The USPSTF recommends against vitamin D supplementation to prevent falls in community-dwelling adults ≥65 years of age in those who are not known to have osteoporosis or vitamin D deficiency (D recommendation).

A previously healthy 34-year-old female presents with a 1-hour history of palpitations. She does not have a cough, shortness of breath, wheezing, or chest pain. She does not take any medications or supplements and does not use alcohol or illicit drugs. She is alert and in no acute distress, but is aware of her fast heart rate. An examination is normal other than the cardiac examination. An EKG is shown below. Which one of the following laboratory tests is most likely to demonstrate the underlying cause of the patient's problem?

A. A BNP level B. A cortisol level C. A lactic acid level D. A troponin level E. A TSH level . . . . . . . . . . . . . . . . . . . . . . . . . . . . . ANSWER: E This patient's EKG shows atrial fibrillation with a rapid ventricular response. A TSH level should be obtained in all patients presenting with acute atrial fibrillation, because even patients with subclinical hyperthyroidism have an increased risk of developing atrial fibrillation. A thyroid abnormality is more commonly the cause of atrial fibrillation in younger age populations because underlying cardiac disease is less common in this age group. A high-sensitivity TSH level is the best initial test to obtain when hypothyroidism or hyperthyroidism is suspected. Additional tests such as free T4 and T3 levels can be ordered when there is a strong suspicion of hyperthyroidism. BNP levels should be ordered if heart failure is suspected. Measuring cortisol levels would not be helpful. An elevated lactic acid level is associated with sepsis. Elevated troponin levels are a diagnostic marker of acute myocardial infarction and cardiac ischemia. Therefore, a troponin level should be obtained when acute coronary syndrome is being considered as a cause of acute atrial fibrillation.

A 55-year-old female presents to your office because she has intermittent locking of her right ring (fourth) finger when it is flexed. It is painful and she often has to use her other hand to extend the finger. Her job involves repetitive movement of her hands and she requests a treatment option that will involve as little missed time from work as possible. Which one of the following would be the most cost-effective option?

A. A course of oral NSAIDs B. A corticosteroid injection C. Splinting of the distal interphalangeal joint D. Physical therapy E. Surgical correction . . . . . . . . . . . . . . . . . . . . . . . . . . . . . ANSWER: B Trigger finger is a common reason for referral to a hand surgeon. Risk factors for this condition include trauma, overuse, diabetes mellitus, and carpal tunnel syndrome. It is much more common in women than in men and the average age of onset is 58. Trigger finger develops when there is scarring and inflammation of the A1 pulley, the first of a five-pulley system in the hand. Stenosis of the A1 canal or nodules on the tendon can produce locking, cracking, and pain when the digit is flexed. The most cost-effective treatment strategy is the use of corticosteroid injections. The success rate is 57% after the initial injection and 86% following the second injection within a 6-month time frame. NSAID injections with diclofenac or ketorolac are also an option and have similar outcomes to corticosteroid injections. When the problem is mild, oral NSAIDs and splinting may be effective. Physical therapy and surgical correction are not indicated for this patient.

A 45-year-old female sees you as a new patient for a health maintenance visit. She has a history of chronic low and midback pain for several years, which she treats with ibuprofen, muscle relaxers, and a heating pad. She takes a combined oral contraceptive pill to control heavy menstrual cycles. She recently went to an urgent care clinic for treatment of an upper respiratory tract infection. She also mentions the gradual appearance of a rash on her back but says that it does not bother her. A skin examination is notable for reticular brown hyperpigmentation of her entire back (shown below). Which one of the following is the most likely cause of the skin changes?

A. Erythema ab igne B. Henoch-Schönlein purpura C. Idiopathic guttate hypomelanosis D. Tinea versicolor . . . . . . . . . . . . . . . . . . . . . . . . . . . . . . . . . . . ANSWER: A Erythema ab igne is characterized by reticular brown hyperpigmented skin changes at the site of exposure to heat, which this patient has developed due to regular use of a heating pad. Exposure to excess heat and humidity can predispose an individual to this condition. Other than removing the precipitating factors, no particular treatment is needed. Henoch-Schönlein purpura is a leukocytoclastic vasculitis and is characterized by palpable purpuric lesions on the lower extremities and buttocks, usually sparing the trunk. It may be preceded by a streptococcal or viral upper respiratory infection. Idiopathic guttate hypomelanosis consists of 2- to 5-mm white spots with sharply demarcated borders on sun-exposed areas of the arms and legs. Tinea versicolor is a fungal infection that is characterized by white, pink, or brown circular macules most commonly located on the upper trunk.

A 5-year-old female is brought to your office with a progressive rash on her legs (shown below) and buttocks. No rash is noted above the level of the mid-torso. Her mother also reports that the child had two episodes of bloody diarrhea 3 days ago. She has abdominal pain and on examination she has abdominal tenderness without rigidity, but with some voluntary guarding. You also note swelling and tenderness in her left wrist and right knee. A CBC, platelet count, prothrombin time, and partial thromboplastin time are normal. A urinalysis reveals mild proteinuria and 5-10 RBCs/hpf. Which one of the following is the most likely diagnosis?

A. Erythema infectiosum (fifth disease) B. Gianotti-Crosti syndrome C. Hemolytic uremic syndrome D. Henoch-Schönlein purpura E. Thrombotic thrombocytopenic purpura . . . . . . . . . . . . . . . . . . . . . . . . . . . . . . . . . . . . . ANSWER: D Henoch-Schönlein purpura is an IgA vasculitis that is usually diagnosed clinically. It is the most common vasculitis in children and occurs less commonly in adults. It presents as palpable purpura of the lower extremities with at least one of the following diagnostic criteria: diffuse abdominal pain, arthritis or arthralgia, renal involvement (hematuria and/or proteinuria), or a skin biopsy showing predominant IgA deposition. Thrombocytopenia and coagulopathy are not present. Henoch-Schönlein purpura is a self-limited condition with an average duration of 4 weeks, and initial treatment is supportive only. Current evidence only recommends corticosteroids for those with more severe disease such as glomerulonephritis. Erythema infectiosum (fifth disease) can be identified by an erythematous rash on the cheeks and a lacy reticular rash on the extremities. Gianotti-Crosti syndrome is a sudden papular or papulovesicular eruption on the extensor surfaces of the arms, legs, buttocks, and face, and it is not purpuric. Hemolytic uremic syndrome presents with the classic triad of hemolytic anemia, thrombocytopenia, and kidney injury. Thrombotic thrombocytopenic purpura is rare in the pediatric age group.

A 54-year-old male sees you for a routine health maintenance examination. He has noticed some fatigue and shortness of breath over the last few months, but he is asymptomatic today. His past medical history is significant for sarcoidosis, which is currently in remission. His vital signs are normal except for an irregular pulse. An EKG performed in the office is shown below. Which one of the following would be most appropriate at this point?

A. Observation B. Amiodarone C. Apixaban (Eliquis) D. Metoprolol succinate (Toprol-XL) E. Referral to a cardiologist for pacemaker placement . . . . . . . . . . . . . . . . . . . . . . . . . . . . . ANSWER: E This patient's EKG shows type II second-degree atrioventricular (AV) block, also called Mobitz type II AV block. Regardless of etiology, Mobitz type II AV block is treated with pacemaker placement because it can lead to complete heart block or sudden cardiac death if left untreated. Conduction disturbances are one of the most common manifestations of cardiac sarcoidosis. In addition to an AV conduction block, as seen in this EKG, other arrhythmias can be seen in those with sarcoidosis. Metoprolol could be used for treatment of some types of tachyarrhythmias. Apixaban is used for anticoagulation in patients with atrial fibrillation or atrial flutter. Amiodarone is indicated for supraventricular or ventricular tachycardias.

A 30-year-old male has moderate to severe rectal pain that occurs with bowel movements and lasts for several hours, persisting for the past 8 weeks. He reports minimal bright red blood on the toilet tissue. On examination he is noted to have an anal fissure in the posterior midline. Sitz baths and fiber supplementation have provided no relief, and topical nifedipine also did not resolve his symptoms. Which one of the following would be an effective treatment? A. β-Blocker injection B. Botulinum toxin injection C. Corticosteroid injection D. Lidocaine injection E. Rubber band ligation

ANSWER : B An anal fissure is a longitudinal tear in the anoderm that most often occurs midline. The initial treatment is sitz baths and increased fiber intake, which leads to resolution in up to 87% of patients. Topical calcium channel blockers such as nifedipine are recommended for patients whose symptoms persist. These medications were found to be better tolerated than topical nitrates with similar healing rates. If there is still no improvement, botulinum toxin injection is indicated. If symptoms continue to persist, surgical therapy may be considered. Neither β-blocker injections nor corticosteroid injections are indicated. Lidocaine would relieve pain but not promote healing and is typically used in topical rather than injection form. Rubber band ligation is indicated for the treatment of internal hemorrhoids, not anal fissures.

A 46-year-old female presents to your office with a 24-hour history of redness in her right eye. It is associated with mild pain but no drainage. On examination her visual acuity is 20/20 bilaterally, her pupillary reflex is normal, her extraocular movements are intact, and there is no discharge noted. There is a focal area of hyperemia of the episcleral blood vessels noted along the medial aspect of the eye. Fluorescein staining is normal. This patient's presentation is most consistent with which one of the following? A. Bacterial conjunctivitis B. Viral conjunctivitis C. Episcleritis D. Iritis E. Keratitis

ANSWER : C An acute red eye is a common presentation in primary care and it is critical to differentiate serious causes from benign causes. Pain is an important distinguishing feature for the acute red eye. Episcleritis is a self-limited condition that can be idiopathic or associated with autoimmune conditions. It is more common in females and usually presents with mild discomfort and focal hyperemia. Discerning episcleritis from scleritis is important. In general, the inflamed vessels of episcleritis will move with gentle pressure from a cotton-tipped applicator whereas those with scleritis will not, and the pain of scleritis is more intense. The treatment of episcleritis is symptomatic with topical lubricants and oral NSAIDs. If recurrent episodes occur, an autoimmune workup should be considered. Conjunctivitis is typically associated with a discharge that is clear in viral cases and mucopurulent in bacterial cases. Iritis is associated with significant pain, a poorly reactive pupil, diminished vision, and photophobia. This patient does not have changes in visual acuity, photophobia, or severe pain as seen in keratitis, which would also cause an abnormal fluorescein stain showing corneal ulceration.

A 25-year-old male presents with a 4-month history of crampy abdominal pain, diarrhea, and fatigue. His symptoms began gradually but have become more severe and he is now experiencing rectal bleeding. He says that his abdominal pain seems to temporarily improve after eating. He has smoked 5-10 cigarettes per day for the past 8 years. He is surprised to learn that he has lost 7 kg (15 lb) when he is weighed today. His vital signs include a blood pressure of 116/70 mm Hg, a heart rate of 76 beats/min, a respiratory rate of 12/min, and a temperature of 37.7°C (99.9°F). A physical examination reveals mild diffuse abdominal tenderness and mild distention. An anorectal examination is significant for a perianal fistula. A laboratory evaluation is notable for mild anemia. His kidney and liver function are normal. Which one of the following is the most likely diagnosis? A. Celiac disease B. Chronic pancreatitis C. Crohn disease D. rritable bowel syndrome E. Ulcerative colitis

ANSWER : C Crohn disease may present insidiously with diarrhea, abdominal pain, rectal bleeding, fever, weight loss, and fatigue. Red-flag symptoms to distinguish inflammatory bowel disease from irritable bowel syndrome include perianal lesions, a first-degree relative with inflammatory bowel disease, weight loss of 5%, abdominal pain for more than 3 months, nocturnal diarrhea, fever, the absence of abdominal pain for 30-45 minutes after eating, and the absence of rectal urgency. This patient exhibits symptoms most consistent with Crohn disease. While anemia is also common in celiac disease, rectal bleeding and a perianal fistula are not. Chronic pancreatitis does not generally present with improved pain after eating. Irritable bowel syndrome is not associated with fever, rectal bleeding, anemia, or perianal fistulas. Ulcerative colitis may present with abdominal pain and rectal bleeding, but it is not associated with perianal lesions.

A 6-month-old male is brought to your office by his mother for a well child examination. The mother does not have any concerns. Interactions between the mother and child are appropriate and the child appears well. Which one of the following screenings is recommended at this visit? A. Autism B. Iron deficiency C. Maternal depression D. Otoacoustic emissions (OAE) testing

ANSWER : C The American Academy of Pediatrics (AAP) recommends formal screening for maternal depression with the Edinburgh Postnatal Depression Scale or the Patient Health Questionnaire-2 (PHQ-2) at the 1-, 2-, 4-, and 6-month well child visits. The U.S. Preventive Services Task Force (USPSTF) recommends screening for depression in all adults, including postpartum patients. It does not offer a specific recommendation or guidance on the timing or frequency of screening. The AAP recommends screening for autism spectrum disorder at 18 months of age, but the USPSTF finds insufficient evidence to recommend routine screening for asymptomatic children. Similarly, the AAP recommends screening for iron deficiency at 12 months, but the USPSTF finds insufficient evidence for universal screening. Otoacoustic emissions testing is recommended during the newborn period and is not recommended at 6 months of age.

An otherwise healthy 18-year-old female comes to your office with a 5-day history of intermittent, sharp chest pain that worsens with deep inspiration and expiration. She has also noticed that the pain worsens with laughing or coughing. It is not present when she is resting quietly. She had symptoms of an upper respiratory infection a week ago with no fever or shortness of breath. COVID-19 testing at that time was negative. She has no history of trauma or prolonged travel and no significant past medical history, and takes no regular medications. Her vital signs are normal, including oxygen saturation, and a physical examination is normal. A chest radiograph and EKG are also normal. Which one of the following would be the most appropriate next step? A. A D-dimer level B. A troponin level C. Spiral chest CT D. Diclofenac, 75 mg twice daily E. Prednisone, 50 mg daily for 5 days

ANSWER : D Ruling out life-threatening conditions is the most important consideration in the evaluation of chest pain. In this case the focus is on pleuritic chest pain. As in all cases of chest pain, a full history and complete physical examination with vital signs should be performed. Tachycardia, tachypnea, hypotension, fever, or respiratory distress should raise concerns. Chest radiography and EKG should be performed if the cause of the pain is unclear. Diagnostic prediction rules such as the MARBURG Heart Score and INTERCHEST clinical decision rule can be utilized to assess the risk of cardiac-related chest pain. With this patient's clear history of pleuritic-type chest pain over 5 days, cardiac ischemia would be very unlikely and a troponin level would not be helpful. If no red flags are raised on history and examination and a chest radiograph and EKG are normal, a trial of NSAIDs is appropriate. NSAIDs are preferable to narcotic medications as they do not suppress respiratory drive and do not have the risk of addiction and abuse. If a life-threatening cause is suspected from the history and physical examination, then further diagnostic testing is indicated. Performing a D-dimer and/or spiral CT would not be useful because the Wells criteria are very low for pulmonary embolism in this patient. Prednisone should not be given unless there are clear reasons why it would benefit the patient.

A 64-year-old male with midsternal chest pain is brought to the emergency department by ambulance. He is on oxygen and an intravenous line is in place. Shortly after arrival he loses consciousness and becomes pulseless and apneic, and CPR is begun. Cardiac monitoring shows ventricular tachycardia with a rate of 160 beats/min. Which one of the following would be most appropriate at this point? A. Amiodarone, intravenous infusion, followed by synchronized cardioversion B. Adenosine, rapid intravenous push, repeated in 1-2 minutes if needed C. Epinephrine, intravenous push, followed by synchronized cardioversion D. Lidocaine (Xylocaine), intravenous push, repeated in 5 minutes if needed E. Defibrillation

ANSWER : E Pulseless ventricular tachycardia (VT) should be treated the same as ventricular fibrillation. The first step is defibrillation. If that is unsuccessful, epinephrine should be administered and defibrillation reattempted. Amiodarone, adenosine, and lidocaine may be used for the initial treatment of wide-complex tachycardia of uncertain type, but should not be used for the initial treatment of pulseless VT. Synchronized cardioversion alone would be indicated for the initial treatment of rapid unstable tachycardia with a pulse.

A 42-year-old right hand-dominant female who owns a bakery presents with a several-month history of gradually worsening pain, swelling, discoloration, and paresthesia affecting her entire right arm. She says that with prolonged use this arm becomes swollen and purplish in color and she develops a deep aching pain. The pain and swelling have become so severe that she is no longer able to make wedding cakes and is concerned she will be unable to continue running her business. She does not recall any trauma and has no swelling in her left arm. She has a history of essential hypertension that is treated with losartan (Cozaar). On examination the patient has full active range of motion and the Neer and Hawkins impingement tests of the shoulder are negative. Based on this patient's history and the physical examination findings, which one of the following is the most likely diagnosis? A. Carpal tunnel syndrome B. Cervical radiculopathy C. Complex regional pain syndrome D. Raynaud disease E. Thoracic outlet syndrome

ANSWER : E This patient's symptoms and physical examination findings are most consistent with thoracic outlet syndrome. This is a condition in which the neurovascular bundle just above the first rib is compressed, leading to a variety of upper extremity symptoms. Thoracic outlet syndrome can be classified by etiology as neurogenic, venous, or arterial, with neurogenic constituting more than 95% of cases. This patient has findings of venous thoracic outlet syndrome: swelling, cyanosis, and distention of superficial veins in the arm suggesting obstruction of the subclavian vein. Paresthesias in the fingers and hand are common and likely due to swelling rather than nerve compression at the thoracic outlet. Due to the exceptionally high risk of developing a venous thrombosis, patients should undergo diagnostic evaluation with upper extremity venous duplex ultrasonography. False negatives are common in patients without a thrombus and in such cases patients may benefit from evaluation with either contrast-enhanced upper extremity CT or magnetic resonance venography. If a thrombus is present anticoagulation should be started immediately and catheterization of the vein should be performed with thrombolysis with or without balloon angioplasty. Ultimately the patient will require surgical decompression.

Which one of the following is considered most useful when implementing shared decision-making? A. Number needed to treat B. Relative risk reduction C. Specificity D. 95% confidence interval E. P-value

ANSWER: A The number needed to treat (NNT) describes the number of patients who receive the intervention and as a result experience the outcome of interest. Many find it to be an intuitive way to explain statistical concepts and to describe the magnitude and direction of benefit from an intervention. Combined with a visual representation of benefit, the NNT is often used with shared decision-making point-of-care tools to help patients visualize their likely benefits and harms from a possible intervention. Relative risk reduction in particular can be misleading, since a seemingly large relative risk reduction may result in a very small absolute risk reduction if the prevalence of the disorder in question is low. Specificity indicates the ability of a test to accurately identify a condition but does not indicate the effectiveness of an intervention. A 95% confidence interval relates to statistical variation and the relationship of an outcome to chance. P-value deals with statistical significance but not magnitude of the effect. Generally, shared decision-making should be offered when a situation of equipoise exists. Examples include a situation where risks and benefits are generally balanced, or where two different approaches are both valid and reasonable paths for management of the situation. This is where patient values and preferences become the most impactful deciding factor in decision-making. Thus, shared decision-making would not be appropriate for instances where a family physician has already concluded that a recommendation is most statistically valid and clinically beneficial.

A healthy 38-year-old female has a few warts on the plantar surface of her great toe. She first noticed them a few years ago after she started to run regularly for exercise and has not tried any at-home treatments. They are minimally raised and rarely painful but occasionally cause irritation and she would like to get rid of them. Which one of the following would be the most appropriate treatment? A. Over-the-counter salicylic acid B. Candida injections C. Application of duct tape D. Manual paring and extraction E. Laser treatment

ANSWER: A A Cochrane review reports good evidence that salicylic acid is effective for the treatment of plantar warts. Candida injections may be indicated for warts that are difficult to treat, but they are not considered a first-line treatment. The application of duct tape has not been shown to be more effective than placebo. Manual paring and extraction of plantar warts carries a greater risk for complications and is not necessary for flat, minimally bothersome warts. Laser treatment may be effective, but the cost is not justified as initial therapy in this case.

A 65-year-old female presents to the emergency department with a 24-hour history of abdominal pain, nausea, and vomiting. She reports that her last bowel movement was 2 days ago and she has not passed any flatus. The pain is diffuse and she rates it as 4 on a scale of 10 but says that it sometimes reaches a level of 8. She has hypertension that is controlled with amlodipine (Norvasc), 5 mg daily. She had an appendectomy at age 25.An examination reveals a blood pressure of 120/80 mm Hg, a pulse rate of 110 beats/min, a respiratory rate of 16/min and nonlabored, and an oxygen saturation of 95% on room air. Examination of the heart and lungs is normal. On examination of the abdomen you note high-pitched bowel sounds. The abdomen is distended, tympanic, and diffusely tender with no appreciable masses. A radiograph of the abdomen is consistent with a small bowel obstruction. Which one of the following is the most likely cause of this patient's small bowel obstruction? A. Adhesions B. Constipation C. A neoplasm D. Ventral herniation E. Volvulus

ANSWER: A Adhesions are the most common cause of small bowel obstruction (SBO) and account for 60%-75% of cases. Less common causes include neoplasms (13%-20% of cases), herniation (2%-15% of cases), and volvulus (<5% of cases). Severe constipation is a rare cause of SBO. The initial management of SBO includes nasogastric tube decompression and intravenous fluid resuscitation. Further evaluation with CT of the abdomen and pelvis is indicated to rule out ischemia or perforation and to determine the etiology of this patient's SBO. Surgical consultation is also recommended. Immediate surgery is indicated for unstable patients, a closed loop obstruction such as volvulus that cannot be reduced, intestinal ischemia, or perforation. Most cases resolve with conservative management, but surgery is recommended if the obstruction has not resolved after 3-5 days.

A 60-year-old male with a 25-year history of diabetes mellitus and peripheral neuropathy sees you because of several recent episodes of feeling faint when assuming an upright posture or with prolonged standing. On examination you ask him to stand from a sitting position, which reproduces his symptoms and causes his blood pressure to drop from 150/100 mm Hg to 90/60 mm Hg. His pulse rate remains in the low 70s. Which one of the following medications would be most likely to exacerbate this patient's symptoms? A. Amitriptyline B. Fludrocortisone acetate C. Midodrine D. Pseudoephedrine E. Epoetin alfa (Procrit, Epogen)

ANSWER: A Amitriptyline is a tricyclic antidepressant (TCA) used for treating neuropathic pain. It has the potential to cause or exacerbate postural orthostatic hypotension. Compared to other TCAs, first-generation TCAs such as amitriptyline are more prone to cause autonomic nervous system dysfunction and dizziness. Fludrocortisone acetate (9-α-fluorohydrocortisone) is used to treat postural hypotension. It causes salt and fluid retention and may cause supine hypertension and hypokalemia. Midodrine is an α-agonist approved to treat postural hypotension. It also can cause fluid retention and hypertension. Pseudoephedrine and ephedrine cause norepinephrine release from presynaptic neurons. Both are α-agonists and have been used to treat postural hypotension, however they may exacerbate supine hypertension. Epoetin alfa is used to treat anemia associated with chronic kidney disease. It causes hypertension, and is unlikely to trigger or exacerbate postural hypotension.

A 14-year-old female is brought to your office after a school screening program identified possible scoliosis. She plays basketball at school and has no history of recent injuries. She is feeling well today and a review of systems is negative. A physical examination reveals an elevated right rib on the forward bend test. Radiography demonstrates a Cobb angle of 15°. Which one of the following would be most appropriate at this point? A. Observation only B. Suspension of sports participation C. Bracing D. Physical therapy E. Surgical evaluation

ANSWER: A Despite a lack of consensus between major health care organizations on the benefit of screening for scoliosis, many states require or recommend school-based screening programs. Adolescent idiopathic scoliosis is generally defined as a lateral curvature of the spine or Cobb angle ≥10°. Cases with a Cobb angle <20° can generally be managed with observation. In this asymptomatic patient there would be no reason to suspend sports participation. Moreover, suspension of sports activity may worsen or contribute to psychologic distress experienced by those with this disorder. In a U.S. Preventive Services Task Force evidence report and systematic review, bracing did decrease progression of the Cobb angle but it did not improve patient-oriented outcomes and did have associated harms. Physical therapy does not have consistent evidence of benefit. Therefore, bracing and physical therapy should be reserved for more severe cases (Cobb 20-39). Surgical evaluation is reserved for severe cases or those with a Cobb angle ≥40°.

You perform the initial newborn examination on a male on his first day of life, following an uncomplicated vaginal delivery at an estimated gestational age of 37 weeks and 6 days. The prenatal course was significant for an initial presentation for prenatal care at 22 weeks gestation. You note that the infant's upper lip is thin and the philtrum is somewhat flat. Which additional finding would increase your concern for fetal alcohol syndrome? A. Curvature of the fifth digit of the hand (clinodactyly) B. A supernumerary digit of the hand C. Flattening of the head (plagiocephaly) D. Metatarsus adductus in one foot E. Syndactyly of the toes (webbed feet)

ANSWER: A Fetal alcohol spectrum disorders are a set of physical, developmental, cognitive, and neuropsychiatric disorders associated with alcohol consumption by the pregnant parent. To be diagnosed with fetal alcohol syndrome, patients must display slow growth, characteristic facial dysmorphia, neurodevelopmental delays (cognitive, behavioral, or both), and central nervous system dysfunction such as microcephaly or seizures. Many children with fetal alcohol syndrome have other neurologic, behavioral, and cardiovascular abnormalities, as well as other types of abnormalities. In addition to clinodactyly, fetal alcohol syndrome is associated with camptodactyly (flexion deformity of the fingers), other flexion contractures, radioulnar synostosis, scoliosis, and spinal malformations. Other findings in children with fetal alcohol syndrome include congenital heart disease, congenital ptosis, ear helix dysmorphia, and midface hypoplasia, among other dysmorphias. Supernumerary digits, plagiocephaly, metatarsus adductus, and syndactyly are common in newborns but are not related to fetal alcohol spectrum disorders.

A 27-year-old female with a history of bipolar disorder with manic episodes sees you because of polyuria and increased thirst over the past month. She has taken lithium, 1800 mg daily, for 3 years and her bipolar disorder is well controlled. She has not lost weight and there is no family history of diabetes. There are no orthostatic blood pressure changes. Laboratory Findings: Serum sodium............145 mEq/L (N 135-145) Serum potassium............4.5 mEq/L (N 3.5-5.0) Serum glucose............92 mg/dL Serum creatinine............0.9 mg/dL (N 0.6-1.5) Serum lithium............1.05 mEq/L (therapeutic range 0.5-1.5) Urine volume............5.15 L/24 hr Urine osmolality............161 mOsm/kg H2O A trial of water restriction is attempted and no change in urine osmolality is seen. Which one of the following is the most likely cause of this patient's problem? A. Medication-induced nephrogenic diabetes insipidus B. Panhypopituitarism C. Prediabetes D. Psychogenic water drinking

ANSWER: A Inappropriately dilute urine (osmolality <300 mOsm/kg) in the setting of hypernatremia suggests diabetes insipidus and, in this case, nephrogenic type where no change in ability to concentrate the urine is seen with water restriction or administration of desmopressin, in contrast to central type in which urine will concentrate after the exogenous administration of desmopressin. Some degree of polyuria occurs ~50% of the time in patients on lithium therapy, both early in therapy and even when plasma lithium levels are in the therapeutic range. Lithium is noted as the most common cause of nephrogenic diabetes insipidus with about 15% of these patients developing the condition as a result of impaired renal concentrating ability resistant to ADH (desmopressin). Patients with hypothalamic or pituitary injuries may develop central diabetes insipidus. In a patient with current use of lithium and no noted history of central nervous system injury, this would not be the most likely cause of the clinical scenario described. The diuresis associated with diabetes mellitus is a result of the osmotic effect of increased serum glucose, which is not present in this case. Psychogenic water drinking may occur in patients with mental health conditions, but would not be expected to cause impairment of renal concentration or hypernatremia as seen in this case.

A 3-year-old child who has not previously received any vaccines is brought to the local health department and diagnosed with measles. Children and staff were exposed to the sick child about 48 hours prior to diagnosis at the day care facility that the child attends. Which one of the following should be given the MMR vaccine as postexposure prophylaxis? A. An otherwise healthy 9-month-old child who is up to date on all age-appropriate immunizations B. A 5-year-old child with asthma who received a second dose of MMR 2 weeks ago C. A pregnant 24-year-old day care staff member who received two doses of MMR as a child D. A pregnant 36-year-old day care staff member who had one dose of MMR vaccine last year when starting work at the day care facility E. A 52-year-old nurse who works part time at the day care facility and received two doses of MMR as an adult

ANSWER: A Measles outbreaks are becoming more common and the CDC has outlined who should receive postexposure prophylaxis with the MMR vaccine. To be effective as postexposure prophylaxis the vaccine must be administered within 72 hours of exposure. Infants <12 months of age are considered to be at high risk for complications from measles and should receive postexposure MMR vaccine, although intramuscular immunoglobulin is also an option. Children who are otherwise fully vaccinated do not need additional prophylaxis. People who are pregnant cannot receive the MMR vaccine due to fetal risk, but they should receive intravenous immunoglobulin if they do not have evidence of immunity. Health care workers only need to be given the MMR vaccine as prophylaxis if they did not receive two doses previously.

A 4-year-old male is brought to your office by his mother for a well child examination. The patient has no significant medical history. The mother has noted that the child has developed new skin lesions over the past few weeks, first appearing on the left arm, with a new lesion behind the right knee. There have been no new detergents or skin or hair care products introduced in the household, nor any changes made in the patient's diet. The child does not have pruritus. The examination reveals a temperature of 37.2°C (99.0°F), a pulse rate of 80 beats/min, and a blood pressure within normal limits. The examination is unremarkable except for non-erythematous flesh-colored, dome-shaped papules with a central indentation that are located on the left lower arm and popliteal fossa. Which one of the following would be most appropriate for the initial management of this common condition? A. Observation only B. Consistent use of emollients and avoiding frequent hot baths C. Use of a topical low-dose corticosteroid cream once daily until resolved D. Use of a topical antifungal cream until resolved E. Paring, followed by topical salicylic acid or cryotherapy

ANSWER: A Molluscum contagiosum is a common disease during childhood that can also occur in adolescents and adults. It is caused by a poxvirus and is uniquely characterized by flesh-colored, dome-shaped papules with central umbilication, most commonly on the trunk, axilla, popliteal or antecubital fossae, and crural folds. If lesions are asymptomatic and not inflamed, the initial treatment is observation, with most lesions resolving spontaneously within 2-12 months. If the lesions are inflamed or pruritic, then topical corticosteroid treatment, chemical treatment with cantharidin, podofilox 0.5% solution, curettage, or cryotherapy may be indicated. Atopic dermatitis (eczema) is initially treated with emollients and by avoiding frequent hot baths. Antifungal cream would be appropriate for tinea corporis but not for molluscum contagiosum. Verruca (warts) are commonly treated with paring, followed by topical salicylic acid or cryotherapy.

A 21-year-old female sees you for a medical evaluation prior to admission to a treatment program for anorexia nervosa. The effects of anorexia on the hypothalamic-pituitary axis include which one of the following? A. Bone loss B. Elevated testosterone C. Hyperglycemia D. Hypothyroidism E. Menorrhagia

ANSWER: A Of the multiple effects anorexia nervosa has on the hypothalamic-pituitary axis, bone loss can be one of the most significant. In a study of 130 women with anorexia nervosa, bone mineral density was reduced by at least 1.0 standard deviation at one or more skeletal sites in 92% of patients. Testosterone levels are often low, contributing to bone loss. Hypoglycemia, not hyperglycemia, can occur but this is not common. Anorexia often results in amenorrhea and infertility due to suppressed estrogen levels. Reduced sex hormone levels and increased cortisol are also thought to result in the bone mineral loss. TSH and T4 levels may be normal or low as a reaction to severe nutritional deficiencies. This state is more appropriately referred to as pseudohypothyroidism rather than hypothyroidism.

Long-term oxygen therapy is recommended for patients with COPD who have a resting oxygen saturation at or below a threshold of A. 88% B. 89% C. 90% D. 93% E. 95%

ANSWER: A Patients with moderate to severe COPD should be evaluated periodically for hypoxemia to determine the need for long-term oxygen therapy. In a Cochrane review of six randomized, controlled trials, oxygen therapy improved survival in select patients with COPD and severe resting hypoxemia (a resting arterial partial pressure of oxygen <55 mm Hg or an oxygen saturation ≤88%). Two classic trials from the 1970s form the basis for long-term oxygen therapy and determine the qualifications for long-term oxygen therapy used by Medicare. Long-term oxygen therapy in patients with exertional hypoxemia and nocturnal hypoxemia has not demonstrated consistently improved patient outcomes.

A 70-year-old female comes to your office for a Medicare annual wellness examination. Her past medical history is notable for controlled hypertension and a successful knee replacement 1 year ago. She mentions that she has a dental appointment to have a cavity filled and asks for your advice on antibiotics to prevent infection in her prosthetic joint. She has no medication allergies or previous history of joint infection and is not immunocompromised. Which one of the following would be the most appropriate recommendation regarding antibiotic prophylaxis for her upcoming dental procedure? A. No antibiotic treatment B. A single dose of oral amoxicillin C. A 3-day course of oral amoxicillin D. A single dose of oral clindamycin (Cleocin) E. A single dose of intravenous cefazolin

ANSWER: A Recent guidelines from the American Dental Association and the American Academy of Orthopaedic Surgeons recommend against the routine use of prophylactic antibiotics for dental procedures in patients with a history of joint replacement, except for situations in which infectious risk is increased, such as immunocompromise or a history of joint infection. This is a change from previous guidelines that recommended routine prophylactic antibiotics prior to dental procedures for patients with prosthetic joints to reduce the risk from the transient bacteremia induced by dental procedures. Infection of the prosthetic joint is a devastating late complication of joint replacement surgery. However, current evidence to support prophylactic antibiotics is limited and antibiotic use is known to increase cost, bacterial resistance, and the risk of adverse medication reactions. In most cases the risks of antibiotic prophylaxis outweigh the likelihood of benefit. Prophylaxis with amoxicillin, clindamycin, or cefazolin would not be appropriate for this patient.

A 60-year-old male with type 2 diabetes comes to your office with an acute onset of fever, chills, and malaise. He says that he is feeling progressively worse. His temperature is 40.0°C (104.0°F). An examination reveals redness, tenderness, and swelling of the penis, scrotum, and perineal area. Which one of the following medications is most likely to predispose this patient to this condition? A. Empagliflozin B. Exenatide C. Insulin glargine D. Pioglitazone E. Sitagliptin

ANSWER: A SGLT2 inhibitors (canagliflozin, dapagliflozin, empagliflozin, and ertugliflozin) are associated with a higher rate of genitourinary infections. Most often these are fungal in etiology, however there are associations with more serious infections including necrotizing fasciitis of the perineum (Fournier's gangrene). While rare, this is a life-threatening infection associated with this class of medication that is being used more frequently to treat diabetes mellitus and other cardiac conditions. Because of this risk, the FDA issued a Drug Safety Warning in 2018. The drug classes that include exenatide, insulin glargine, pioglitazone, and sitagliptin are not associated with genitourinary infections.

A 42-year-old female presents for follow-up after recurrent visits for gradually increasing shortness of breath, cough, and a decrease in her usual activity level over the past year. She does not have a history of smoking, secondhand smoke exposure, or occupational exposures. She reports feeling a little better after a short course of oral prednisone and use of an albuterol (Proventil, Ventolin) inhaler. You review the radiology report, which describes panlobular basal emphysema. Spirometry in the office reveals an FEV1/FVC ratio of 0.67 with no change after bronchodilator administration. Which one of the following underlying conditions is the most likely cause for this clinical presentation? A. α1-Antitrypsin deficiency B. Bronchiectasis C. Diffuse panbronchiolitis D. Interstitial lung disease E. Left heart failure

ANSWER: A Spirometry in this patient confirms airflow limitation or obstruction with an FEV1/FVC <0.7. Her age, the lack of tobacco smoke or other occupational exposures, and the chest radiograph findings of emphysematous changes at this early stage are typical of α1-antitrypsin deficiency. While bronchiectasis, diffuse panbronchiolitis, interstitial lung disease, and left heart failure are all causes of chronic cough, they are not necessarily associated with the development of COPD and these spirometry findings. Furthermore, the radiologic findings in this patient are not consistent with these conditions. Bronchiectasis would present with bronchial dilation and bronchial wall thickening on a chest radiograph. Diffuse panbronchiolitis would present with diffuse small centrilobular nodular opacities along with hyperinflation. Interstitial lung disease would generally present with reticular or increased interstitial markings in later stages and with a restrictive rather than obstructive pattern on spirometry. Left heart failure would present with pulmonary edema on a chest radiograph and volume restriction on pulmonary function testing.

A 25-year-old gravida 1 para 0 at 28 weeks' gestation comes to your office for a routine prenatal visit. She received Tdap vaccine 3 years ago after she was bitten by a horse. Which one of the following would be most effective to reduce the newborn's risk for contracting pertussis? A Tdap vaccination of the mother now B. Tdap vaccination of the mother now, and Tdap vaccination of the newborn within 72 hours of birth C. Tdap vaccination of the mother post partum D. No Tdap vaccination of the mother now or post partum, and Tdap vaccination of other family members E. No Tdap vaccination of the mother now or post partum, and a recommendation for good hand hygiene to protect the newborn

ANSWER: A The Advisory Committee on Immunization Practices recommends Tdap vaccination during each pregnancy regardless of the time interval since the last booster, primarily to confer immunity against pertussis to the infant. To maximize the concentration of pertussis antibody transferred to the fetus, administration of the vaccine is recommended between 27 and 36 weeks' gestation. Postpartum vaccination of the mother with Tdap does not provide as much protection as vaccination during pregnancy. Newborns should not receive Tdap but should begin pertussis immunization at 2 months of age with the first dose of DTaP. There is no recommendation to vaccinate infants earlier to reduce the risk of vertical transmission. Family members and close contacts should be vaccinated against pertussis at least 2 weeks prior to contact with the infant, but this strategy alone is less effective than maternal immunization. Good hand hygiene is important when caring for an infant, but on its own is not an effective strategy to reduce the risk of pertussis transmission.

In the United States, which one of the following groups has the highest prevalence of syphilis? A. Men who have sex with men B. Men who have sex with women C. Women who have sex with men D. Women who have sex with women E. Women who have sex with women and men

ANSWER: A The risk for syphilis infection is highest among men who have sex with men and among persons who are HIV-positive. In 2021 the CDC reported 176,713 cases of syphilis in the United States, an increase of 73% from the previous year. It is thought that disruptions in people's lives and in the health care system due to the COVID-19 pandemic account for this change. The USPSTF recommends that asymptomatic, nonpregnant adults and adolescents who are at increased risk, as well as all persons who are pregnant, should be screened for syphilis infection (A recommendation). In the United States the prevalence of syphilis is also increased in young adult males and in persons with a history of incarceration, sex work, or military service. Specific neighborhoods and areas with a higher social deprivation index may also have higher rates of syphilis infection. The USPSTF recommends that local community and socioeconomic factors be considered when identifying patients at increased risk for infection who should be screened.

A 20-month-old female is brought to your office because of a cough she has had for the last 5 days. She had a fever only on the first day of the illness. On examination the child is active and no respiratory distress is noted. The ENT and pulmonary examinations are normal. She has tested negative for respiratory syncytial virus and COVID-19. No other persons in her home are ill. Her caregiver is wanting recommendations for something to alleviate the child's cough. Which one of the following treatments for cough has evidence of efficacy and safety when used for children from 1 to 2 years of age? A. Honey B. Dextromethorphan C. Diphenhydramine (Benadryl) D. Codeine E. Guaifenesin

ANSWER: A The use of over-the-counter or prescription medications to treat symptoms of upper respiratory infection in children under 4 years of age has come under scrutiny in recent years due to lack of effectiveness and evidence of potential harms (SOR B). In 2008, the FDA recommended against the use of over-the-counter cough and cold medications for children younger than 2 years due to concerns about efficacy and safety. Soon thereafter, the American Academy of Pediatrics recommended avoiding all cough and cold medication in children younger than 6 years. However, symptomatic relief can be achieved using effective treatments for cold symptoms in children, including nasal saline irrigation, menthol rub, and honey, all of which have been shown to be safe and effective in children over the age of 12 months (SOR B). Two Cochrane reviews and one randomized, controlled trial demonstrated the effectiveness of honey in reducing the frequency and severity of cough in children. It should be avoided in children younger than 1 year of age due to the risk of botulism, but is safe in children 1 year of age or older. Recommendations for dosing include 2.5 mL for children 2-5 years of age, 5 mL for children 6-11 years of age, and 10 mL for children 12-18 years of age. Safety and/or efficacy data in children under 2 years of age is not available for dextromethorphan, diphenhydramine, codeine, and guaifenesin. Codeine in particular has serious safety problems in young children and should be avoided.

A 55-year-old male with a 40-pack-year smoking history comes to your office with the results of spirometry he had at a health fair. He quit smoking 1 year ago. He does not have any cough, dyspnea, wheezing, or sputum production, but he is concerned that the spirometry results show an FEV1/FVC ratio of 0.65 and an FEV1 of 70% of predicted, which indicates mild to moderate airflow obstruction. Based on the best available evidence, which one of the following should you recommend in order to prevent the development of symptomatic airflow obstruction? A. No treatment B. A long-acting anticholinergic C. A long-acting β-agonist D. An inhaled corticosteroid E. Combination therapy with a corticosteroid and long-acting β-agonist

ANSWER: A There is no evidence from randomized, controlled trials to show that treating asymptomatic individuals who have mild to moderate airflow obstruction on spirometry prevents future respiratory symptoms or reduces subsequent declines in lung function. Partly for this reason, the U.S. Preventive Services Task Force and joint guidelines issued by the American College of Physicians, American College of Chest Physicians, American Thoracic Society, and European Respiratory Society recommend against screening for COPD in asymptomatic adults. Regardless of the results of this patient's spirometry testing, treatment should not be initiated in the absence of symptoms. Detailed history-taking would be appropriate to detect patients who have limited their activity in order to avoid symptoms. A long-acting anticholinergic, a long-acting β-agonist (LABA), an inhaled corticosteroid (ICS), and combination therapy with an ICS and a LABA would not be recommended for this patient.

A 59-year-old male presents with pain over the lateral distal aspect of his right foot. He states that he accidentally struck a filing cabinet with his foot about 5 days ago. He immediately had pain and swelling over the dorsum of his foot and his second to fifth toes, and there has been no improvement. He has tried buddy taping his toes but this has not provided any relief. A physical examination is notable for trace edema over the dorsum of the right foot and tenderness to palpation over the distal fifth metatarsal and fifth phalanx. The foot is neurovascularly intact with a strong dorsalis pedis pulse and normal sensation. A radiograph of the right foot is shown below. In addition to pain control, which one of the following would be most appropriate? A. A rigid-sole shoe and weight bearing as tolerated B. A pneumatic brace and weight bearing as tolerated C. A short leg walking cast and weight bearing as tolerated D. A posterior splint and no weight bearing E. Immediate referral to an orthopedic surgeon and no weight bearing

ANSWER: A This patient has a minimally displaced oblique right fifth proximal phalanx shaft fracture. Lesser toe fractures such as this typically can be managed conservatively with buddy taping and a rigid-sole shoe. Because this patient has already tried buddy taping, it is appropriate to recommend a rigid-sole shoe. Pneumatic braces can be used for some nondisplaced tuberosity avulsion fractures. Metatarsal shaft fractures are often initially treated with a posterior splint and then transitioned to a walking cast. Referral to an orthopedic surgeon is typically limited to patients with a high-risk fracture such as a displaced Jones fracture, or to patients who are highly competitive athletes.

A 29-year-old male presents with a 10-day history of a cough. Early in the illness, he also had a low-grade fever for 2 days that has since resolved. He has had some mild rhinorrhea and states that the cough has produced occasional greenish sputum mostly in the mornings over the past 3-4 days. He has not had any blood in the sputum. He smokes one pack of cigarettes per day normally, but stopped smoking when he became ill. He has not tried any medications. He had two negative COVID-19 tests at home. An examination reveals normal vital signs and some mild rhinorrhea but the lung examination is normal. Which one of the following would be most appropriate at this point? A. Supportive care only B. Albuterol (Proventil, Ventolin) C. An inhaled corticosteroid D. Antibiotic therapy E. A chest radiograph

ANSWER: A This patient has findings consistent with acute bronchitis. Even in patients who smoke the etiologic agent is viral at least 90% of the time, so antibiotics are not routinely indicated. Persons who smoke are prescribed antibiotics more frequently, but there is little to no evidence that this practice changes the disease course of acute bronchitis. A 2017 Cochrane review concluded that antibiotics have only very slight benefit in the mean duration of cough of less than 1 day over a 10-day period in otherwise healthy persons, and their use is more commonly associated with negative side effects. Unless wheezing is noted, albuterol is not helpful. Inhaled corticosteroids are used for asthma. Indications for a chest radiograph in an adult patient with acute bronchitis include: bloody sputum, rust-colored sputum, or dyspnea; a pulse rate >100 beats/min; a respiratory rate >24/min; or a temperature >37.8°C (100.0°F). A chest radiograph is also indicated if there are abnormal findings on a chest examination such as fremitus, egophony, or focal consolidation. Supportive care is made easier by informing the patient that symptoms are likely to last 2-3 weeks. Symptoms may be managed with measures such as dextromethorphan, guaifenesin, or honey. Patients with diagnosed COPD may benefit from antibiotics when given for a COPD exacerbation. Utilization studies note that patients with private insurance receive antibiotics at higher rates, and White patients receive antibiotics at higher rates than other races or ethnicities.

A 68-year-old female with a history of diabetes mellitus, hypertension, and heart failure with preserved ejection fraction presents for management of a diabetic foot ulcer that has worsened. Her medications include the following: Atorvastatin (Lipitor), Furosemide (Lasix), Insulin glargine (Lantus), Lisinopril (Zestril), Metformin A CBC reveals a WBC count of 7600/mm3 (N 4500-11,000), a hemoglobin level of 9.7 g/dL (N 14.0-17.5), a mean corpuscular volume of 89 µm3 (N 80-100), and a platelet count of 412,000/mm3 (N 150,000-400,000). To further assess the patient's anemia you obtain the following laboratory results: Ferritin............293 ng/mL (N 22-275) Serum iron............43 µg/dL (N 50-175) Transferrin............190 mg/dL (N 177-264) Reticulocyte count............3.2% (N 0.5-1.5) Vitamin B12............564 pg/mL (N 230-1050) Haptoglobin............198 mg/dL (N 63-273) Which one of the following is the most likely cause of her anemia? A. Anemia of chronic disease B. Bone marrow suppression C. Hemolysis D. Iron deficiency E. Vitamin B12 deficiency

ANSWER: A This patient has findings most consistent with anemia of chronic disease, also known as anemia of inflammation. This condition is thought to be primarily a disorder of iron distribution in response to systemic inflammation, which also biases hematopoiesis toward myeloid cell production rather than erythropoiesis and shortens the erythrocyte lifespan. Anemia of chronic disease is a normocytic and normochromic anemia. Iron studies typically show evidence of iron restriction without systemic iron deficiency. A common challenge in diagnosis is when true iron deficiency coexists with anemia of chronic disease. Her history of a chronic foot ulcer and elevated inflammatory markers (ferritin and platelets) are consistent with anemia of chronic disease. This patient's normal WBC and platelet counts make bone marrow suppression less likely. The normal haptoglobin level and a normal reticulocyte index calculated for this degree of anemia are not consistent with hemolysis. She has a normocytic rather than microcytic anemia and her ferritin level is elevated. These two factors make iron deficiency less likely despite her low serum iron level. The low normal transferrin level is also consistent with anemia of chronic disease rather than iron deficiency. Her normal vitamin B12 level makes a deficiency unlikely.

An 85-year-old female is admitted to the hospital for surgery and develops confusion postoperatively. The patient is a widow and lived independently prior to admission. Her daughter says the confusion is atypical for her mother. She does not have a history of memory loss, forgetfulness, or confusion prior to admission. Which one of the following cognitive assessment tests should be used to assess her acute change in mental status? A. Confusion Assessment Method (CAM) B. Mini-Cog C. Mini-Mental State Examination (MMSE) D. Montreal Cognitive Assessment test (MoCA) E. Saint Louis University Mental Status exam (SLUMS)

ANSWER: A This patient is experiencing an acute cognitive change from baseline, indicating possible delirium. The Confusion Assessment Method (CAM) is a delirium diagnostic tool useful for evaluating acute cognitive changes. The Mini-Cog, Mini-Mental State Examination, Montreal Cognitive Assessment, and Saint Louis Mental Status exam are designed to test chronic baseline cognitive function and do not test for acute changes

An otherwise healthy 46-year-old female presents with a 10-day history of recurring pain in the right cheek and gums. She says the pain feels like electric shocks lasting a few seconds and recurs "hundreds of times a day." She says that smiling and brushing her teeth can trigger the pain. She does not have a history of recent dental work, trauma, fever, or myalgia. She feels well aside from the facial pain. She has tried over-the-counter analgesics without relief. Her vital signs and an HEENT examination are normal. There is no pain with dental percussion, and the skin and mucous membranes of the nose and mouth are unremarkable. Which one of the following would be the most appropriate treatment for this patient's condition? A. Carbamazepine (Tegretol), 200 mg twice daily B. Prednisone, 40 mg daily C. Sumatriptan (Imitrex), 6 mg subcutaneously D. Valacyclovir (Valtrex), 1000 mg three times daily E. High-flow oxygen for 20 minutes

ANSWER: A Trigeminal neuralgia is a clinically diagnosed condition that is characterized by brief, sudden, unilateral pain in the distribution of one of the three branches of the trigeminal nerve. The pain, which is often triggered by minimal stimulus, is paroxysmal with episodes of remission that can last for months. First-line treatment for the condition is carbamazepine or oxcarbazepine. Other suggested treatments include lamotrigine, baclofen, and surgical treatments, including microvascular decompression. Short-term corticosteroids are not recommended for trigeminal neuralgia. Sumatriptan is an appropriate treatment for migraines that are also unilateral but are associated with photophobia, phonophobia, nausea, and persistent severe pain. Valacyclovir would be an appropriate treatment for herpes zoster, which may cause unilateral electrical shock-like pain in the face. However, patients with herpes zoster typically would have developed the characteristic lesions by 10 days. In addition, the pain is rarely only brief and paroxysmal in nature with herpes zoster. High-flow oxygen is used to relieve cluster headaches that occur in the orbital, temporal, or supraorbital areas and persist for 15-180 minutes. Cluster headaches are accompanied by tearing and nasal discharge.

A 42-year-old female sees you for follow-up after a motor vehicle accident. CT of the abdomen performed at the time of the accident demonstrated an incidental finding of several uterine fibroid tumors measuring <5 cm. She currently has no abdominal or pelvic pain, and she reports regular menstrual cycles. Which one of the following would be the most appropriate management option? A. Observation only B. An oral contraceptive C. A GnRH agonist D. Laparoscopic myomectomy E. Hysterectomy

ANSWER: A Uterine fibroid tumors (leiomyomas) are the most common tumors of the female reproductive tract, with some evidence suggesting that the cumulative incidence in women ages 25-45 is approximately 30%. Symptoms related to fibroids can include menorrhagia, pelvic pain, obstructive symptoms, infertility, or pregnancy loss. However, many fibroids are asymptomatic and are discovered incidentally, with observation being the preferred management in this situation. The risk of leiomyosarcoma is exceedingly small and there is a risk of complications from other treatment modalities. For women who are symptomatic, the data is insufficient regarding the most appropriate therapy. Surgical options include myomectomy, hysterectomy, uterine artery embolization, and myolysis, but data to allow direct comparison is lacking. With the exception of trials of GnRH-agonist therapy as an adjunct to surgery, there is not enough randomized trial data to support the use of medical therapies such as oral contraceptives, NSAIDs, or progestins in the treatment of symptomatic fibroids.

A 62-year-old male with hypertension presents to your office with substernal chest pain radiating into his left arm for the past 20 minutes. He also has diaphoresis and nausea. He has a blood pressure of 156/96 mm Hg, a pulse rate of 84 beats/min, and an oxygen saturation of 93% on room air. An EKG shows ST-segment elevations in leads V1 and V2. Your medical assistant calls 911 for immediate transport to the local hospital emergency department. While awaiting the ambulance's arrival you Qgive the patient aspirin, 325 mg. Which one of the following would be most appropriate regarding oxygen therapy at this time? A. No oxygen therapy B. Oxygen via nasal cannula at 2 L/min C. Oxygen via nasal cannula at 6 L/min D. 100% oxygen with a regular mask E. 100% oxygen with a nonrebreathing mask

ANSWER: A While oxygen supplementation is routinely initiated for patients who are suspected of having acute coronary syndrome, evidence does not support a benefit from this unless the patient is hypoxic. Oxygen supplementation is only recommended if the patient has an oxygen saturation <90%, if the patient is at risk for hypoxemia, or if the patient is in respiratory distress. There is also some evidence that hyperoxygenation in this setting may be harmful.

A healthy 49-year-old female presents to your office for a routine health maintenance visit. Since her last visit a year ago she has had only two menstrual periods. She reports sudden sensations of extreme heat in her face, neck, and chest that last just a few minutes but occur throughout the day. These symptoms are very bothersome and interfere with the quality of her sleep. After noting that her medical evaluation does not reveal any increased risks, you discuss potential therapies. Which one of the following would you recommend as the most effective option to help relieve her symptoms? A. Black cohosh B. Combined estrogen and progesterone C. Compounded bioidentical hormones D. Micronized progesterone E. Testosterone

ANSWER: B .This patient presents with typical vasomotor symptoms due to menopause. Vasomotor symptoms affect 80%-90% of women, and symptoms can persist a decade or more. Symptoms often begin early in perimenopause and become more frequent after the cessation of menses. They are known to occasionally have substantial effects on sleep quality and are associated with an increased risk of cardiovascular and skeletal health conditions. Combination estrogen and progesterone therapy is highly effective for vasomotor symptoms. The estrogen component relieves symptoms and the progesterone component minimizes risks of endometrial hyperplasia and cancer. Both are necessary in any patient who still has a uterus in order to avoid endometrial hyperplasia and increased risk of endometrial cancer. The evidence to support the effectiveness of black cohosh has been inconsistent. Compounded bioidentical hormone therapy creates safety concerns and is not a first-line therapy due to limited regulation, the potential for overdosing and underdosing, impurities or lack of sterility, and the lack of labeling describing risks. Although micronized progesterone decreases vasomotor symptoms there are no long-term studies to assess the safety of progestin-only treatment for menopausal symptoms. Testosterone alone has not been shown to reduce vasomotor symptoms. It may be useful for hypoactive sexual desire in postmenopausal women, but it is not approved by the FDA for use in women.

A 29-year-old mother of three young children asks about the benefits of giving probiotics to her children. You tell her that probiotics have been shown to decrease A. diarrhea associated with irritable bowel syndrome B. the likelihood of antibiotic-associated diarrhea C. the development of allergies D. colic symptoms in formula-fed infants

ANSWER: B A growing body of research has established the benefits of probiotics, especially in children. Probiotics, particularly Saccharomyces boulardii, have been shown to prevent the antibiotic-associated diarrhea that occurs in 5%-30% of children who receive antibiotics (SOR A). The number needed to treat to prevent one case of diarrhea is 10, and improves to six when using high-dose probiotics (≥5 billion colony-forming units per day). Probiotic use is associated with reduced pain in children with irritable bowel syndrome (IBS) but has not been shown to be helpful in reducing diarrhea or constipation in those patients. In a meta-analysis of six randomized, controlled trials involving breastfed infants with colic, Lactobacillus reuteri was associated with reduced daily crying time (average 56 minutes). Similar benefits have not been found in formula-fed infants or infants who are combining breastfeeding and formula. Probiotics have not been shown to prevent colic in infants. Other conditions with some evidence of benefit with probiotic use include eczema and upper respiratory infections (SOR A). However, their use has not been shown to prevent allergies or asthma.

A 46-year-old runner presents with left heel pain that has been occurring mostly while running, but more recently also with walking. On examination there is tenderness and a palpable nodule on the mid-substance of the left Achilles tendon. Which one of the following is the best therapeutic option for initial treatment? A) Oral NSAIDs B) Eccentric calf muscle contraction exercises C) Corticosteroid injection of the Achilles tendon sheath D) Surgical debridement or excision of the tendon nodule E) Immobilization in a walking boot for 30-60 days

ANSWER: B Achilles tendinopathy is among the most common injuries in middle-aged distance runners, and the most effective treatment overall is exercise rehabilitation. Achilles tendon injuries can be separated into insertional tendinopathy (about 25% of injuries), midportion tendinopathy (about 50% of injuries), and proximal musculotendinous junction (about 25% of injuries). Recovery from Achilles tendinopathy may take up to a year, especially if treatment is delayed or reinjury occurs. Various Achilles tendon loading programs are recommended. Complete rest can be detrimental and prolong recovery, and immobilization is not recommended. Historically, eccentric muscle contractions have been used, but protocols involving isolated concentric or combination concentric and eccentric contractions have also produced benefit. More recently, isometric exercise has been recommended as the initial treatment for tendinopathy, so contraction and movement of the muscle seems to be the key. Oral NSAIDs may be helpful for temporary pain relief, but they contribute little to recovery from this injury. Corticosteroid injection into the peritendinous space has little evidence of benefit and is associated with a risk of tendon rupture. Surgical debridement may be considered as a last resort for difficult cases.

A 56-year-old male with a history of hepatitis C cirrhosis is admitted to the hospital with GI bleeding. The patient has been stable, taking only furosemide and spironolactone. Upper GI endoscopy confirms variceal bleeding and the gastroenterologist performs appropriate variceal banding. A nurse calls you because laboratory studies ordered in the emergency department reveal a serum ammonia level of 120 µg/dL (N 39-90). The patient has no signs of confusion, insomnia, or decreased mental alertness. A physical examination reveals mild ascites but no other abnormalities. Which one of the following would be most appropriate for addressing the elevated ammonia level? A. Lactulose B. No additional treatment C. Methotrexate D. Neomycin E. Prednisone

ANSWER: B Elevated ammonia levels may occur in multiple clinical scenarios (i.e. portosystemic shunting, UTI from urease-producing organisms, GI bleeding, shock, renal disease, parenteral nutrition, salicylate intoxication, alcohol use). In patients with chronic liver disease, hepatic encephalopathy is diagnosed based on the overall clinical presentation and not by an ammonia level. It is important to remember that a normal ammonia level neither excludes nor confirms the diagnosis of hepatic encephalopathy. This patient had an elevated serum ammonia level that was found incidentally during his hospital admission for gastrointestinal bleeding. Because there is no clinically significant encephalopathy, treatment based on ammonia levels is not indicated. Lactulose, methotrexate, neomycin, or prednisone would not be appropriate.

A 60-year-old male comes to your office with a 1-year history of a gradual onset of fatigue and dyspnea on exertion. He notes no chest pain or heaviness with exertion. He has a 20-pack-year history of cigarette smoking but quit at age 38. He has a BMI of 30 kg/m2 with normal vital signs. A physical examination is unremarkable. Office spirometry reveals a decreased FVC at 73% of predicted and a normal FEV1/FVC ratio, and there are no changes after bronchodilator administration. Which one of the following would you recommend at this point? A. The 6-minute walk test B. Complete pulmonary function testing C. Bronchoprovocation testing such as a methacholine challenge test D. A ventilation-perfusion scan E. Bronchoscopy

ANSWER: B Family physicians are often required to manage dyspnea and evaluate common office spirometry results. The American Thoracic Society recommends complete pulmonary function testing with diffusing capacity of the lungs for carbon monoxide (DLCO) measurement when office spirometry suggests a restrictive pattern, which is defined as an FVC less than 80% of predicted. This level of pulmonary function testing gives further information about gas exchange and lung volumes, allowing a more definitive diagnosis. The 6-minute walk test is used to evaluate the treatment response for known cardiopulmonary disease. Bronchoprovocation testing helps identify asthma triggered by allergens or exercise when office spirometry is normal. Bronchoscopy is an invasive test that is not indicated at this point in the evaluation. A ventilation-perfusion scan may be used in the evaluation for suspected pulmonary embolus, but it is not the preferred diagnostic test in the evaluation of chronic dyspnea in a patient with normal vital signs.

You see a 3-year-old female with a 2-day history of intermittent abdominal cramps, two episodes of emesis yesterday but none today at this time, and about five watery, nonbloody stools each day. She does not have a fever, her other vital signs are normal, and she has not traveled recently. Today her parent reports that she has tolerated sips of fluid and still seems more tired than usual. An examination is normal except for mildly dry lips. A friend at preschool had a similar illness recently. Which one of the following would be the most appropriate initial management of oral intake in this patient? A. A sports drink and food on demand B. Half-strength apple juice and food on demand C. Ginger ale and no food yet D. Water and no food yet E. A bolus of intravenous normal saline and no food yet

ANSWER: B Family physicians often see patients with diarrheal illnesses and most of these are viral. Patients sometimes have misconceptions about preferred fluid and feeding recommendations during these illnesses. The World Health Organization recommends oral rehydration with low osmolarity drinks (oral rehydration solution) and early refeeding. Low osmolarity solutions contain glucose and water, which decrease stool frequency, emesis, and the need for intravenous fluids. Soda and sports drinks contain a higher concentration of glucose, which may worsen diarrhea. Half-strength apple juice has been shown to be effective, and it approximates an oral rehydration solution. Its use prevents patient measurement errors and the purchase of beverages with an inappropriate osmolarity. It is also more appealing to children than many oral rehydration solutions. Water increases the risk of hyponatremia in children. This patient is not ill enough to need intravenous fluids. Refeeding on patient request has been shown to decrease the duration of illness.

A 45-year-old G3P2012 comes to your outpatient clinic for a routine health maintenance examination. She reports mild left pelvic discomfort over the past few weeks with no other symptoms. She is otherwise healthy and has an unremarkable family history. She has been having regular periods and is sexually active with her husband, who had a vasectomy 9 years ago. On examination you notice a fullness in the left adnexal region that is mildly tender to palpation. The remainder of the examination is normal. You order pelvic ultrasonography, which demonstrates a 4×5-cm cystic structure on the left ovary. There are no internal septations or echogenic internal structures. A pregnancy test is negative. Which one of the following would be most appropriate at this point? A. Reassurance only B. Repeat pelvic ultrasonography in 8-10 weeks C. A serum CA-125 level D. An oral contraceptive and repeat pelvic ultrasonography in 6-8 months E. Referral to a gynecologist for diagnostic cystectomy

ANSWER: B Functional ovarian cysts are estimated to be present in approximately 15% of menstruating women and are often found on diagnostic imaging performed for other reasons. The majority of these lesions resolve spontaneously within 2-3 menstrual cycles and management should consist of follow-up ultrasonography 8-12 weeks after the cyst is identified. In a Cochrane study that included eight randomized, controlled trials, the use of oral contraceptives to treat functional ovarian cysts appeared to be of no benefit in reducing the time to cyst resolution. While widely used for treatment of this condition, there is no evidence for their benefit. Watchful waiting for 2-3 cycles is appropriate. When cysts have features that are inconsistent with simple cysts (<5 cm, unilocular), persist for more than 8-12 weeks, or are >10 cm in size, referral should be made for consideration of diagnostic removal (cystectomy or oophorectomy depending on patient characteristics). Patients who carry a BRCA gene or who have a family history of ovarian cancer should also be considered for invasive testing sooner. Serum CA-125 testing is difficult to interpret in premenopausal patients, who frequently have elevated levels without evidence of a malignancy.

You are preparing printed educational materials for patients, families, and caregivers. Based on current recommendations, these materials should be written at or below what reading level? A. First to second grade B. Fifth to sixth grade C. Eighth to ninth grade D. Tenth grade E. Twelfth grade

ANSWER: B It is recommended that all printed patient education information be written at or below a fifth- to sixth-grade reading level. Even though the average education level in the United States is much higher, the grade-level reading ability of the population is lower and textual materials need to be written at an appropriate level for patients. Many studies that analyze published patient education materials have found them to be at significantly higher reading levels than fifth to sixth grade. These literacy precautions are recommended by professional organizations such as the American Medical Association and the Agency for Healthcare Research and Quality.

A 48-year-old female presents to the emergency department and is diagnosed with a pulmonary embolus following CT angiography. Which one of the following initial findings would be the strongest indication for urgent thrombolytic therapy? A. Elevated troponin B. Hypotension C. Hypoxia D. Bilateral pulmonary emboli E. Right ventricular dysfunction on echocardiography

ANSWER: B Patients with high-risk (massive) pulmonary embolism (PE) include those who have experienced hemodynamic decompensation with hypotension, cardiogenic shock, or cardiac arrest. Recent guidelines have suggested that patients with hypotension (a systolic blood pressure <90 mm Hg or a diastolic blood pressure <60 mm Hg, for 15 minutes or longer) should be treated with reperfusion therapy with thrombolysis if they are not at high risk for bleeding, or with other interventions aimed at rapid reversal such as surgical pulmonary embolectomy. Intermediate-risk (submassive) PE describes normotensive patients with evidence of right ventricular compromise or an elevated troponin level, whereas low-risk PE patients are hemodynamically stable, have low risk factor scoring via the Pulmonary Embolism Severity Index (PESI) or other risk calculator, and have neither evidence of an elevated troponin level or right ventricular dysfunction.

A 55-year-old male presents to your clinic for evaluation of shortness of breath and abdominal pain of 6 weeks' duration. He has a history of type 2 diabetes, coronary artery disease, and untreated hepatitis C. He works a physical job at a steel plant, and reports smoking one pack of cigarettes and drinking three cans of beer daily. A chest radiograph shows a significant right-sided pleural effusion. Thoracentesis reveals a pleural fluid protein to serum protein ratio of 0.3, and a pleural LDH to serum LDH ratio of 0.4. The pleural glucose level is 140 mg/dL and his serum glucose level is 150 mg/dL. Which one of the following is the most likely cause of this pleural effusion? A. Asbestosis B. Cirrhosis with ascites C. Malignancy D. Pneumonia E. Viral illness

ANSWER: B Pleural fluid analysis is essential in the evaluation of a new pleural effusion. Standard testing includes protein, glucose, LDH, pH, Gram stain and culture, and cytology. These results can help determine if the effusion is exudative or transudative by using Light's criteria: a pleural fluid protein to serum protein ratio >0.5, a pleural fluid LDH to serum LDH ratio >0.6, or a pleural fluid LDH >0.67 times the upper limit of normal for serum LDH suggests an exudative effusion. Lower ratios suggest a transudative process. Additionally, transudative pleural fluid pH is typically between 7.40 and 7.55, with fewer than 1000 WBCs, and the glucose level is similar to the serum glucose level. Cirrhosis with ascites and heart failure are two common causes of transudative effusion. Pleural effusions associated with asbestosis, malignancy, pneumonia, and viral illness tend to be exudative.

A 12-hour-old newborn is seen in the nursery for routine newborn care. The prenatal course and delivery were unremarkable. An examination is normal except for a single 1-cm wide dimple on the sacrum, 1 cm superior to the anus. The dimple has a tuft of dark hair. At this point you would recommend: A. routine follow-up at the 12-month well child check B. ultrasonography C. MRI D. a fistulogram/sinogram E. a dermatology consultation

ANSWER: B Recognizing clinically significant abnormalities on the newborn examination is important. One of the more serious conditions to assess for during a newborn examination is occult spinal dysraphism (OSD), a term that encompasses a spectrum of congenital anomalies involving the spine and/or surrounding structures. Early detection and treatment of OSD can prevent long-term irreversible neurologic impairment. A sacral dimple can be a sign of OSD. Newborns with small (<5 mm) sacral dimples located within 2.5 cm of the anal verge, and without other skin findings such as hair, do not need imaging to rule out OSD because there is very low incidence of abnormal findings. If one dorsal midline skin lesion is seen, then meticulous assessment for other skin lesions is warranted to determine whether two or more dimples are present, as this increases the risk of OSD. The dimple described here does not meet the low-risk criteria described, which makes imaging necessary in this case. Ultrasonography is recommended as the first-line imaging test. It can accurately detect spinal dysraphism and does not require sedation. A pediatric neurosurgery consultation would be indicated if the ultrasound examination was abnormal. MRI is the next step in evaluation and should be considered initially for patients with an abnormal neurologic examination or multiple dimples, which increase the risk of OSD, but would not be necessary in this case. A follow-up examination without imaging would be unacceptable at this time given the findings described and the urgency of early diagnosis. The optimal timing of the ultrasound examination is debatable and some studies have found improved accuracy of ultrasonography at a corrected gestational age of 42.5 weeks compared to ultrasonography performed within a couple days of birth. If there are no neurologic deficits, this timing is safe and appropriate.

A 42-year-old male with a history of alcohol use disorder was admitted to the hospital last night with mid-epigastric pain and tenderness. Findings from the hospital evaluation included an elevated lipase level and a normal gallbladder and biliary tree ultrasound examination. He was diagnosed with acute pancreatitis, placed on NPO status, and started on intravenous fluids and pain control. This morning he still has moderate epigastric pain and tenderness, and mild nausea but no vomiting. He says he would like to try eating some food. Which one of the following would you recommend at this time? A. Continuing NPO status until the pain and tenderness have improved B. Allowing the patient an oral low-fat diet as tolerated C. Initiating nasogastric feeding D. Initiating nasojejunal tube feeding E. Initiating total parenteral nutrition

ANSWER: B The American Gastroenterological Association guidelines on acute pancreatitis recommend initiating oral feedings within 24 hours if tolerated in patients with mild pancreatitis. Early feeding, which is thought to play a role in protecting the gut-mucosal barrier, may limit infectious complications and does not increase hospital length of stay or other complications. Early feeding is also associated with a reduction in pancreatic necrosis and organ failure when compared with delayed feeding. Nasogastric or nasojejunal tube feeding may be considered at 3-5 days if oral feedings are not tolerated. Total parenteral nutrition is indicated only when enteral feedings cannot supply adequate caloric intake or are not possible for other reasons. The incidence of single or multiple organ failure or infected necrosis is significantly increased with the use of total parenteral nutrition.

A U.S. hospital or birthing center seeking to be certified as "Baby-Friendly" by the Baby-Friendly Hospital Initiative must satisfy which one of the following criteria in addition to meeting other requirements? A. Demonstrating proper use of an infant car seat to parents prior to discharge B. Providing no other food or fluids to breastfeeding infants without a medical indication C. Providing a pacifier to each baby prior to discharge D. Providing easy access to a variety of infant formulas E. Providing on-site daycare facilities for staff

ANSWER: B The Baby-Friendly Hospital Initiative is a global program established by UNICEF and WHO to promote healthy infant feeding and mother-baby bonding. One of the key components of the program is that breastfeeding infants should not be given food other than breast milk, unless medically indicated. The primary objective of the initiative is to educate the public on the benefits of breastfeeding and have medical and nursing staff encourage, promote, and facilitate breastfeeding as outlined in the UNICEF/WHO Ten Steps to Successful Breastfeeding. These steps promote breastfeeding to the public and provide guidelines for hospitals and birthing centers for the successful initiation and continuation of breastfeeding. Baby-friendly facilities must have a written breastfeeding policy that is routinely communicated to all maternity unit health care staff, and staff must be trained in the skills necessary to implement this policy. All pregnant patients should be informed about the benefits and management of breastfeeding. Patients should be helped to initiate breastfeeding within an hour after birth and shown how to breastfeed and to maintain lactation, even if they are separated from their infants. Other aspects of the program are that rooming in should be practiced, allowing mothers and infants to remain together 24 hours a day. Mothers should be encouraged to breastfeed on demand. Breastfeeding infants should not be given pacifiers or artificial nipples. Mothers should be referred to breastfeeding support groups on discharge from the hospital. In addition, the hospital must comply with the International Code of Marketing of Breast Milk Substitutes, which requires that formula companies cannot give free gifts to staff or mothers, that breast milk substitutes are not marketed in the maternity unit, and that breast milk supplements and infant feeding supplies are purchased at fair market price.

Intensive behavioral intervention has been shown to be more effective than other treatment modalities in treating children who have been diagnosed with: A. attention-deficit/hyperactivity disorder B. autism C. depression D. obsessive-compulsive disorder E. posttraumatic stress disorder

ANSWER: B The only evidence-based treatment that confers significant benefit to children with autism is intensive behavioral intervention, for at least 25 hours per week, which should be initiated before 3 years of age. These interventions, delivered over an extended time frame, improve cognitive ability, language, and adaptive skills. Attention-deficit/hyperactivity disorder, depression, obsessive-compulsive disorder, and posttraumatic stress disorder can be treated with behavioral therapies but pharmacologic therapy is often required.

A 4-year-old male is brought to your office by his maternal aunt, who is his new guardian. She is concerned that he is exhibiting problems with behavior and attention. On examination you note long, wide, protruding ears, an elongated face, and frontal bossing. Which one of the following is the most likely cause of these dysmorphic features? A) Angelman syndrome B) Fragile X syndrome C) Klinefelter syndrome D) Marfan syndrome E) Prader-Willi syndrome

ANSWER: B The prepubescent male child with fragile X syndrome can be recognized by large ears, an elongated face, macrocephaly, or frontal bossing. Other features of fragile X syndrome include increased risk for chronic otitis media, esotropia, hyperextensible finger joints, high-arched palate, low muscle tone, and, occasionally, seizures. This presentation can be subtle in young children, with an average age at diagnosis of 8 years. After puberty, a prominent jaw and macro-orchidism are characteristic. Behavioral aspects seen may include poor eye contact, excessive shyness, anxiety, hand flapping, hand biting, aggression, tactile defensiveness, attention deficits, hyperactivity, impulsivity, hyperarousal to sensory stimuli, and autism spectrum disorder. Facial dysmorphic features associated with Angelman syndrome include microbrachycephaly, maxillary hypoplasia, a large mouth, and prognathism. Prepubescent boys with Klinefelter syndrome do not have facial dysmorphic features. They appear similar to prepubescent boys with normal karyotypes. Although a child with Marfan syndrome has an elongated face, the frontal bossing and large ears are not characteristic of that condition. Facial dysmorphic features associated with Prader-Willi syndrome include a narrow distance between the temples, almond-shaped eyes, and a thin upper lip.

A 65-year-old female comes to your office for her Welcome to Medicare visit. Her current medication regimen includes losartan (Cozaar), 50 mg daily for hypertension; oxybutynin extended release, 5 mg daily for overactive bladder; doxepin, 10 mg at night for depression and insomnia; and rosuvastatin (Crestor), 10 mg daily for hyperlipidemia. She is doing well on her current medications and is requesting refills today. You advise her that the benefits of her current regimen for her overactive bladder and depression need to be weighed against her increased risk of A. cirrhosis B. dementia C. Parkinson's disease D. peripheral vascular disease E. pulmonary fibrosis

ANSWER: B The use of anticholinergic medications and other sedatives has been prospectively linked to an increased risk of dementia. Oxybutynin and doxepin both have anticholinergic properties, and the potential risks of these medications should be discussed with this patient. Alternative medications are often available and should be considered for older patients when possible. There is no association between the use of anticholinergic agents and cirrhosis, Parkinson's disease, peripheral vascular disease, or pulmonary fibrosis.

A 54-year-old male comes to your office to re-establish care. He has a past history of hypertension treated with lisinopril (Zestril) and hydrochlorothiazide but has not taken his medications for over a year because his employer closed the factory where he worked and he lost his health insurance. He does not have any chest pain, shortness of breath, or headache. On examination his average blood pressure is 200/115 mm Hg on three separate readings taken 5 minutes apart. The remainder of the physical examination is normal. Which one of the following management options would be most appropriate? A. Institute out-of-office monitoring with an ambulatory device and follow up in 2 weeks B. Restart the patient's previous antihypertensive medications and follow up within 1 week C. Administer a short-acting antihypertensive medication in the office to lower his blood pressure to <160/100 mm Hg D. Hospitalize for hypertensive emergency

ANSWER: B This patient has severe asymptomatic hypertension (systolic blood pressure ≥180 mm Hg or diastolic blood pressure ≥110 mm Hg). Because this patient is asymptomatic and has a known history of hypertension, restarting his prior antihypertensive regimen and following up within 1-2 weeks would be the most appropriate management option. If there were signs or symptoms of acute target organ injury, such as neurologic deficits, altered mental status, chest pain, shortness of breath, or oliguria, hospitalization for a hypertensive emergency would be indicated. In the absence of acute target organ injury, blood pressure should be gradually lowered to <160/100 mm Hg over several days to weeks. Aggressively lowering blood pressure can lead to adverse events such as myocardial infarction, cerebrovascular accident, or syncope, so administering a short-acting antihypertensive medication in the office should be reserved for hypertensive emergencies.

A 28-year-old female comes to your office at 37 weeks gestation with a 24-hour history of painful vesicles on the vulva. She does not have a past history of similar lesions. You make a presumptive diagnosis of genital herpes. Of the following, the most sensitive and specific test is: A. exfoliative cytology (Tzanck test) B. a polymerase chain reaction (PCR) test of vesicular fluid C. an enzyme-linked immunosorbent assay (ELISA) D. HSV serology (IgG/IgM)

ANSWER: B When genital herpes occurs during pregnancy, the best method of diagnosis is a PCR test of the fluid from a vesicle, which is sensitive and very specific. ELISA are sensitive, but not as sensitive or specific as PCR. The Tzanck test is also less sensitive and less specific than PCR testing. Serology is generally not helpful. Patients without a history or current symptoms should not be screened with either genital swabs or serology. The rate of false-positive tests approaches 50% and previous trials have not supported reduction of risk to the parent or infant through screening.

A fully immunized 9-month-old male is brought urgently to your office after his parents noted a possible seizure. His mother says that the infant began to "shake all over" for about 3-4 minutes and then promptly fell asleep for about 20 minutes. When he awoke he was alert but fussy and crying. He has been ill for the last few days with a cough, congestion, elevated temperature, and decreased oral intake. Just prior to the shaking, he had a fever of 101°F. On examination he has a temperature of 38.3°C (100.9°F), a heart rate of 170 beats/min, a respiratory rate of 50/min, and an oxygen saturation of 97% on room air. The infant is fussy but consolable. His mucous membranes are moist, his tympanic membranes are clear, and he has a normal oropharynx. He has clear rhinorrhea. Examination of the heart is normal, and examination of the lungs reveals rhonchi and wheezes. He is moving all of his extremities normally. Which one of the following would be the most appropriate initial step in the evaluation of this child? A. A basic metabolic panel B. Radiography of the chest C. MRI of the brain D. Electroencephalography E. A lumbar puncture

ANSWER: B This child's history and physical examination are most consistent with a simple febrile seizure. Characteristics of a simple febrile seizure include short duration (less than 15 minutes), generalized tonic-clonic movements (as opposed to focal), no postictal pathology (such as Todd paresis), and no recurrence within 24 hours. They occur most often from age 6 months to 6 years and are associated with a low risk of developing epilepsy (1 in 40). Children who have a simple febrile seizure should be examined and evaluated to determine the source of their fever. This child has signs of possible pneumonia so a chest radiograph would be warranted to look for the source of infection. Routine metabolic laboratory testing is not indicated in the workup of simple febrile seizures in children. There is a low risk that these children will have low sodium or glucose levels, and this would not predict seizure recurrence. Neuroimaging such as MRI is not recommended for an isolated simple febrile seizure. Electroencephalography is not useful for predicting the recurrence of simple febrile seizures and would not be indicated in the workup of these seizures. A lumbar puncture is indicated only in cases with neurologic findings suggestive of meningitis, but that is not the case for this child.

Which one of the following is the most effective measure to protect children from lead toxicity in the United States? A. Anticipatory guidance for parents and caregivers during well child visits B. Checking the serum lead level after a known exposure C. Eliminating the sources of lead in the community D. Iron and calcium supplementation to reduce lead absorption E. Providing appropriate cleaning equipment to families with known lead in the home

ANSWER: C Although lead poisoning in children has decreased over the past few decades, it is still a problem in the pediatric population. The most effective way to protect U.S. children from lead toxicity is primary prevention, which includes reducing or eliminating the sources of lead in the community. Anticipatory guidance regarding hand washing or dust control, checking serum lead levels after exposures, iron and calcium supplementation, and providing cleaning equipment have been shown to have either little or no effect, or they address high lead levels only after the lead poisoning has occurred.

While examining a 65-year-old male with a history of rheumatic valvular disease you hear a heart murmur. Which one of the following is the principal auscultatory finding of aortic regurgitation? A. An S3 gallop heard best at the cardiac apex with the patient supine B. A triphasic pericardial friction rub with the patient seated and leaning forward C. A low-pitched decrescendo diastolic murmur that is loudest at the lower left sternal border with the patient seated and leaning forward D. A high-pitched crescendo/decrescendo midsystolic murmur that is loudest at the right upper sternal border and radiates to the carotid arteries with the patient sitting upright E. A harsh holosystolic murmur that is loudest at the lower left sternal border and radiates to the left lateral chest wall with the patient in the left lateral decubitus position

ANSWER: C Aortic regurgitation may occur due to a congenital bicuspid aortic valve, which is often accompanied by the systolic murmur of aortic stenosis. Rheumatic aortic valvular disease may cause aortic stenosis and aortic regurgitation, which is the most common etiology in the developing world but less common in the United States. The hallmark murmur of aortic regurgitation is a "blowing" decrescendo diastolic murmur along the tract from the aortic valve (upper right sternal border) down to the lower left sternal border, where it is loudest. It is best heard with the patient sitting and leaning forward with breath held in expiration. The other murmurs characterized above are systolic murmurs of aortic or mitral disease or descriptions of other heart or pulmonary sounds that could be encountered in other patients.

A 63-year-old male sees you after carotid ultrasonography at a local health fair showed a 50% occlusion of his left proximal internal carotid artery. He has never had a TIA or stroke and he takes no medications. He smokes one pack of cigarettes daily. His blood pressure today is 122/76 mm Hg. In addition to counseling on a healthy diet, exercise, and tobacco cessation, you should recommend: A. no further treatment or follow-up B. observation, and repeat ultrasonography in 1 year C. statin therapy, and repeat ultrasonography in 1 year D. statin therapy and referral to a vascular surgeon for consideration of a carotid artery stent E. statin therapy and referral to a vascular surgeon for consideration of carotid endarterectomy

ANSWER: C Asymptomatic carotid stenosis of 50%-70% should be treated medically to reduce ASCVD. Initiating statin therapy and smoking cessation will have the greatest impact on reducing his risk. Without past neurologic symptoms from carotid artery disease, surgical intervention would not be indicated due to the potential risks of surgery, but repeat ultrasonography is considered reasonable. Extracranial carotid artery atherosclerosis is a risk factor for stroke and warrants risk factor reduction. Nonsurgical approaches that lower stroke risk include smoking cessation, hypertension management with a goal blood pressure of <130/80 mm Hg, management of diabetes mellitus if present, and initiation of high-intensity statin therapy with a target LDL-cholesterol level of <70 mg/dL. Regular physical activity regardless of BMI is associated with a reduction in stroke risk. Surgical interventions for carotid disease, such as endarterectomy or stenting, carry a significant risk of mortality and disabling stroke. Because of the risks of intervention, screening of asymptomatic patients is not routinely recommended. Similarly, the benefits of intervention must be weighed against the potential risks. Generally, intervention is recommended if the patient has experienced significant symptoms from carotid artery disease such as stroke or high-risk TIA with stenosis >50%. For asymptomatic patients, the average annual risk of stroke with stenosis ≥70% is about 1% and evaluation for carotid endarterectomy or stenting is recommended if the patient is considered to be at acceptable surgical risk.

A 20-year-old female with a history of persistent depression sees you for follow-up. She reports unplanned weight loss, chronic insomnia, fatigue, a loss of interest in hobbies that she previously enjoyed, and decreased concentration. She has seen other physicians and has tried antidepressant medications, including paroxetine (Paxil), bupropion (Wellbutrin), and citalopram (Celexa). She reports that there was a week when her mood felt much more normal. She needed only about 3 hours of sleep each night during that week. She admits she was somewhat impulsive and experienced racing thoughts at that time. She does not personally use alcohol or drugs, but has several family members who have a substance use disorder. She has a family history of suicide in her mother and older sister. A physical examination, laboratory studies, and drug screen are all unremarkable. Which one of the following is most likely to help this patient? A. Escitalopram (Lexapro) B. Mirtazapine (Remeron) C. Quetiapine (Seroquel) D. Trazodone E. Venlafaxine (Effexor XR)

ANSWER: C Bipolar disorders often present in late childhood or early adolescence. Outcomes can be improved by early recognition. Manic episodes that occur with bipolar I disorder are usually easy to identify. However, patients with bipolar II disorder, such as this patient, may have a hypomanic episode that goes unrecognized, and the patient may present with persistent depression. There is usually a family history of bipolar disorder or multiple relatives having persistent depression, obsessive-compulsive disorder, attention-deficit disorder, or panic disorder. There may be instances in the family of suicide, substance use disorder, alcohol use disorder, or incarceration. There are many options for the treatment of bipolar disorders. Quetiapine is a reasonable choice that can be used for acute mania, for bipolar disorder with mixed features, and as maintenance therapy. Other medications with high-quality evidence to support their use in bipolar disorder include lithium, valproic acid, lamotrigine, cariprazine, lurasidone, and antipsychotics

A 64-year-old female presents to the emergency department with a 10-day history of increasing shortness of breath and mild tachycardia. On examination she has an oxygen saturation of 75% on room air. Which one of the following additional findings would suggest a diagnosis of acute respiratory distress syndrome (ARDS)? A. Improved oxygen saturation with supplemental oxygen B. Improvement of her symptoms with diuretic therapy C. Bilateral airspace opacities seen on a chest radiograph D. A flattened diaphragm seen on a chest radiograph E. A right lower lobe infiltrate seen on a chest radiograph

ANSWER: C Chest radiographic findings in ARDS include bilateral airspace opacities but not a localized infiltrate as with pneumonia or a flattened diaphragm as with COPD. Clinically, ARDS will often present similarly to pneumonia or heart failure with dyspnea, hypoxemia, and tachypnea. However, ARDS typically does not respond to supplemental oxygen or diuretic therapy. Patients may decompensate quickly and require mechanical ventilation. The Berlin classification required mechanical ventilatory support in the definition of ARDS, but the COVID-19 pandemic has demonstrated that some patients with mild ARDS can be treated with noninvasive respiratory support.

A 30-year-old male is treated with topical medications for his papulopustular rosacea with only partial improvement. He has no drug allergies. The preferred oral antibiotic in this situation is: A. amoxicillin B. cephalexin (Keflex) C. doxycycline D. erythromycin E. sulfamethoxazole/trimethoprim (Bactrim)

ANSWER: C Doxycycline (100-200 mg daily or 40 mg once daily of a modified-release formulation) and minocycline (100-200 mg daily) are effective options for the treatment of papulopustular rosacea. The modified-release doxycycline, which is a 40-mg capsule, is FDA-approved but is more expensive out of pocket. Oral metronidazole or macrolides such as azithromycin and clarithromycin can also be considered for those who cannot take tetracyclines. Erythromycin would not be a first-line choice. Amoxicillin, cephalexin, and TMP-SMX lack evidence to support their use in the treatment of papulopustular rosacea.

During a newborn examination you are asked by the parents if there is anything they can do to decrease the risk for food allergies in their infant. There is no family history of atopic dermatitis or asthma but they note a first cousin with a peanut allergy. You tell them that food allergy risk can be reduced by: A. avoiding a day care setting B. breastfeeding for at least 1 year C. introducing peanut-containing foods when solids are started D. using soy-based formula instead of cow's milk-based formula

ANSWER: C Food allergy affects 8% of children in the United States. IgE-mediated food allergy is the best understood, and symptoms can range from rhinorrhea to anaphylaxis. The three most common allergens are eggs, cow's milk, and peanuts. Symptoms typically occur within 2 hours of exposure and resolve within several hours. Until 2008, various guidelines recommended delaying the introduction of allergenic foods, such as peanuts, until 3 years of age. The National Institute of Allergy and Infectious Diseases currently recommends that healthy infants without known food allergy may have peanuts introduced freely into their diet along with other solid foods around 4-6 months of age. Those with mild to moderate eczema may be introduced to peanut-containing foods around 6 months of age. Infants with severe eczema, known egg allergy, or both should undergo peanut-specific allergy testing before introducing peanuts at 4-6 months of age. While breastfeeding may decrease atopic disease, there is insufficient evidence that it reduces the likelihood of food allergy, and using a soy-based formula will not prevent food allergy. Likewise, breastfeeding, using hydrolyzed formulas, and avoiding cow's milk-based formula does not appear to reduce the risk of cow's milk protein allergy. However, supplementation using cow's milk-based formula in the first week of life may actually increase the risk of cow's milk allergy. Introducing portions of well-cooked egg around 4-6 months of age probably reduces the risk of chicken egg allergy. There is no data that avoiding day care will reduce food allergies.

A 40-year-old male sees you for a routine health maintenance examination. You discuss screening for hepatitis C virus (HCV) infection, as he has not had this done previously.According to the U.S. Preventive Services Task Force, routine HCV screening is recommended for A. all patients with a history of hepatitis A virus infection B. all patients with a history of hepatitis B virus infection C. all patients 18-79 years of age regardless of history D. only patients born between 1945 and 1965 E. only high-risk individuals with a history of transfusions or intravenous drug use

ANSWER: C Hepatitis C virus (HCV) is the most common chronic bloodborne pathogen. According to a 2020 update by the U.S. Preventive Services Task Force, routine HCV screening is recommended for all patients 18-79 years of age regardless of history. This recommendation does not require patients to have a history of engaging in high-risk behavior or a history of hepatitis A or hepatitis B. Routine screening was previously recommended only for patients born between 1945 and 1965.

A 70-year-old male is being treated with metformin for type 2 diabetes. He is still employed and is otherwise in good health. Which one of the following hemoglobin A1c values is associated with the lowest mortality in this situation? A. 5.0%-5.9% B. 6.0%-6.9% C. 7.0%-7.9% D. 9.0%-9.9%

ANSWER: C In patients ≥65 years of age treated with medication for type 2 diabetes, hemoglobin A1c values of 7%-8% have shown the greatest reduction in mortality in multiple studies. The specific hemoglobin A1c target between 7% and 8% should be based on shared decision-making and the overall condition of the patient at that specific age, with goals in the lower 7% range for those with good to excellent functional status. It is suggested that lower hemoglobin A1c values are associated with frequent hypoglycemia, which presents a greater risk than a higher hemoglobin A1c value alone. Hemoglobin A1c values over 9% are associated with greater mortality (SOR B). Thus, while the risk of complications increases linearly with hemoglobin A1c, mortality has a U-shaped curve. Management of blood pressure and treatment with statins improves mortality in these patients as well and is important in addressing overall cardiovascular risk.

A 46-year-old female with a past medical history of polycystic ovary syndrome and migraine headaches presents with bilateral, hyperpigmented patches along her mandible. The patches are asymptomatic but bother her cosmetically and seem to be darkening. Which one of her medications would be most likely to contribute to her melasma? A. B-complex vitamins B. Metformin C. Oral contraceptives D. Spironolactone (Aldactone) E. Sumatriptan (Imitrex)

ANSWER: C Melasma is a progressive, macular, nonscaling hypermelanosis of skin exposed to the sun, typically involving the face and more rarely the dorsal forearms. It is often associated with pregnancy and the use of oral contraceptives or anticonvulsants. Although thought previously to involve only activation of melanocytes, it is now seen as a complex syndrome involving an interplay of keratinocytes, dermal mast cells, gene regulation, and vascular changes. Women are more likely to be affected than men, and darker-skinned individuals with Fitzpatrick skin types III-IV are more commonly affected. There are three common patterns of melasma that are described: centrofacial, malar, and mandibular, although most clinical cases are not clearly one type only. Initial treatment of melasma may include sun protection and discontinuation of any provoking medications. Traditionally, melasma has been treated with topical agents such as hydroquinone, tretinoin, and corticosteroids, or combinations of these agents. Tranexamic acid has gained popularity as a systemic therapy for melasma and is available in oral, topical, and injectable forms.

A 65-year-old male with known metastatic lung cancer is hospitalized after presenting with a 2-week history of decreased appetite, lethargy, and confusion, and a laboratory evaluation that revealed the following: Serum calcium............15.8 mg/dL (N 8.4-10.0) Serum phosphorus............3.9 mg/dL (N 2.6-4.2) Serum creatinine............1.6 mg/dL (N 0.7-1.3) Total serum protein............5.0 g/dL (N 6.0-8.0) Albumin............3.1 g/dL (N 3.7-4.8) Which one of the following would be the most appropriate INITIAL management? A. Calcitonin-salmon (Miacalcin) subcutaneously B. Pamidronate by intravenous infusion C. Normal saline intravenously D. Furosemide intravenously

ANSWER: C The initial management of hypercalcemia of malignancy includes fluid replacement with isotonic fluids. Increased serum calcium levels stimulate urinary calcium excretion and increase urinary output, which in turn causes volume depletion. A reasonable regimen, in the absence of edema, is the administration of isotonic saline at an initial rate of 200 to 300 mL/hour that is then adjusted to maintain the urine output at 100 to 150 mL/hour. Hypercalcemia can also cause vomiting, diarrhea, decreased oral intake, and acute kidney injury, which can all contribute to dehydration. Restoring intravascular volume with isotonic fluids to achieve a urine output of at least 100 mL/hr enhances renal calcium excretion. Although intravenous pamidronate has become the mainstay of treatment for hypercalcemia of malignancy, it should be considered only after the patient has been made euvolemic by isotonic fluid repletion. The same is true for calcitonin. The use of loop diuretics such as furosemide should be considered only if signs of fluid overload are present, since these drugs can aggravate volume depletion and are not very effective alone in promoting renal calcium excretion.

A 75-year-old male presents to your office with a growth (shown below) that has developed under his eye over the past several months. He says the growth is painless but appears to be slowly enlarging. Which one of the following is the most likely diagnosis? A. Actinic keratosis B. Fibrous papule C. Nodular basal cell carcinoma D. Sebaceous hyperplasia E. Seborrheic keratosis

ANSWER: C The pearly, papular appearance of this patient's growth and the overlying telangiectasia makes the most likely diagnosis a nodular basal cell carcinoma. Basal cell carcinoma is the most common cutaneous malignancy. The incidence increases with age and occurs most commonly in Fitzpatrick skin types 1 and 2. The tumors appear most frequently on the face, scalp, ears, and neck, and less frequently on the torso and extremities. In-office dermoscopy may make the branching blood vessels or telangiectasias characteristic of basal cell carcinoma easier to see. In pigmented basal cell carcinomas, dermoscopy can highlight pigmented globules or other areas of abnormal deposition not easily seen without a dermatoscope. Basal cell carcinoma can be locally destructive but rarely metastasizes. Actinic keratoses are characterized by rough patches with a slight scale that form gradually. Sebaceous hyperplasia begins as elevated papules that eventually become dome-shaped and umbilicated. It consists of small tumors made up of sebaceous glands. Fibrous papules are a common variant of angiofibromas. They are usually 1-5 mm in diameter and most often appear on the nose. They do not typically grow once they have appeared. Seborrheic keratoses are benign skin neoplasms that are tan or black, well circumscribed, and have a stuck-on appearance.

An adult male sees you for follow-up of type 2 diabetes and hyperlipidemia. Routine laboratory findings include an AST level of 73 U/L (N 10-59) and an ALT level of 88 U/L (N 13-40) with normal alkaline phosphatase and total bilirubin levels. These levels are persistently elevated 3 months later. He states that he drinks 1-2 glasses of wine per week and does not use acetaminophen. His medications include metformin and atorvastatin (Lipitor). Additional laboratory testing shows negative serology for viral hepatitis and a normal transferrin saturation. An ultrasound demonstrates evidence of hepatic steatosis. Which one of the following would be most appropriate at this point? A. Discontinue atorvastatin B. Discontinue metformin C. Calculate a Fibrosis-4 score D. Order CT of the abdomen E. Refer for a liver biopsy

ANSWER: C This asymptomatic patient with mildly elevated transaminases most likely has nonalcoholic fatty liver disease (NAFLD), which is the most common chronic liver disorder in the United States. It is associated with metabolic syndrome. The initial evaluation should include studies to rule out less common causes of liver disease including viral hepatitis and hemochromatosis. Other laboratory studies that assist in evaluation include albumin and platelet levels. These values allow for the calculation of the Fibrosis-4 score or the NAFLD fibrosis score, which are validated to predict the risk of significant liver fibrosis. Patients with an elevated risk of fibrosis require further evaluation, typically with ultrasound-based elastography before considering liver biopsy. Medications and supplements may cause elevated transaminase levels, and a thorough history to elicit this information is important. Statin-induced liver injury is rare and not consistent with this clinical picture. Discontinuing statin therapy is not necessary with mild transaminase elevations due to NAFLD. Metformin is unlikely to cause elevated transaminases and is safe with this severity of liver disease. Liver fibrosis may be detected with CT but ultrasonography is more sensitive and thus preferred.

An 18-year-old male is brought to the urgent care clinic by his friends several hours after he cut his hand when he punched someone in the mouth during a fight involving several people. On examination he is alert and shows no signs of distress. He has multiple superficial scratches on his arms and legs, swelling over his left eye, and a 5-mm laceration over the third metacarpophalangeal joint of his right hand with intact sensation and motor function and no active bleeding. There is no appreciable erythema. In addition to assessing the current status of this patient's Tdap immunization, which one of the following would be the most appropriate treatment of his hand injury? A. Irrigation of the wound alone B. Irrigation of the wound followed by suturing of the laceration C. Irrigation of the wound, application of dressing, and initiation of PO Augmentin 875 mg BID for 3-5 days, with follow-up in 24 hours D. Application of dressing and follow-up in 24-36 hours E. Admission to the hospital for intravenous antibiotics and orthopedic consultation

ANSWER: C This patient has a clenched-fist bite wound. Without antibiotics these wounds have a high rate of infection, and irrigation alone is not sufficient. Because this patient has a superficial wound with no evidence of tendon involvement or functional compromise, he does not require ongoing parenteral antibiotics and can be managed as an outpatient. Small wounds should be allowed to heal by secondary intention, so suturing is not appropriate. Larger wounds may need to be explored for tendon injuries or other structural damage.

A 14-year-old female is brought to your office as a new patient for a routine well child examination. She feels well and does not take any medications. Her past medical, surgical, family, and social histories are unremarkable. A review of systems is notable for no history of menstruation. An examination is notable for a height at the first percentile with a normal BMI and breast development at Tanner stage 1. Laboratory studies reveal an elevated FSH level. Which one of the following would be the most appropriate next step? A. Follow-up every 3-6 months for assessment of pubertal development B. A corticotropin stimulation test C. Karyotyping D. Radiography of the hand for bone age E. MRI of the brain and pituitary

ANSWER: C This patient has delayed puberty, as evidenced by Tanner stage 1 breast development at age 14. Underlying etiologies should be excluded in females without breast development by age 13 or in males without testicular enlargement by age 14. In addition to a thorough history and a physical examination, a gonadotropin measurement should be performed as part of the initial evaluation. Constitutional growth delay is the most common cause of delayed puberty in males and females and is a diagnosis of exclusion. A family history of delayed puberty is common in these cases. The elevated FSH level in this patient indicates hypergonadotropic hypogonadism. When taken together with her short stature, Turner syndrome is an important consideration. Turner syndrome is characterized by a lack of normal X chromosome gene expression (45,X karyotype) and is estimated to occur in 1/3000 births. Females with Turner syndrome have ovarian sex hormone insufficiency, which leads to delayed puberty. Delayed diagnosis of Turner syndrome is common, and short stature and delayed puberty are sometimes the only symptoms. Diagnosis is made via karyotyping. A corticotropin stimulation test could be considered in the setting of precocious puberty to rule out Cushing syndrome. Radiography of the hand for bone age may support a finding of delayed growth (and thus support treatment with growth hormone for a patient with Turner syndrome), but would not provide valuable diagnostic information in this scenario. MRI of the brain is not indicated with an elevated FSH level that is consistent with an appropriately functioning hypothalamus and pituitary gland.

A 73-year-old female is brought to your office by her daughter, who is concerned that there may be some memory changes in her mother. She has noticed that her mother frequently repeats herself and has made several medication errors lately. The patient has type 2 diabetes, hypertension, depression, and hypothyroidism. Her current medications include the following: A. Aspirin B. Atorvastatin (Lipitor) C. Glyburide D. Hydrochlorothiazide E. Lisinopril (Zestril) F. Metformin G. Sertraline (Zoloft) A physical examination reveals a blood pressure of 136/72 mm Hg and is otherwise normal. She scores 26/30 on a Saint Louis University Mental Status (SLUMS) examination, suggesting mild cognitive impairment. A Geriatric Depression Scale screening is normal. Laboratory studies reveal a hemoglobin A1c of 7.0% and a TSH level of 3.8 µU/mL (N 0.4-4.2). A CBC and basic metabolic panel are normal and her glucose level is 93 mg/dL. Which one of her medications should be stopped in this patient?

ANSWER: C This patient has signs of mild neurocognitive impairment. Correct medication adherence is important, especially with medications that can cause hypoglycemia, as that can lead to confusion and delirium. The sulfonylurea glyburide carries a risk of significant hypoglycemia, especially in elderly patients. Glyburide, in particular, is listed on the Beers Criteria for potentially inappropriate medication use in older adults because of its potential to cause prolonged hypoglycemia. This patient is at the age and level of functional status where tight control of diabetes is no longer recommended, and her hemoglobin A1c goal should be relaxed. A statin such as atorvastatin should be continued in diabetic patients unless there are severe side effects. Lisinopril and hydrochlorothiazide are unlikely to be causing these neurologic side effects with a normal serum potassium level and a normal creatinine level. Inadequate treatment of depression may cause pseudo-dementia (a dementia-like presentation due to depression), but her Geriatric Depression Scale screening is negative. Her greatest risk of a life-threatening medication side effect would come from her oral hypoglycemic medication.

A 68-year-old female presents with a 3-month history of low back pain and fatigue. She has experienced an unintentional weight loss of 15 lb. A physical examination is positive for vertebral point tenderness over the third and fourth lumbar vertebrae. A complete neurologic examination is normal. Initial laboratory studies reveal a normocytic anemia, an elevated total serum protein level, and a mild decrease in renal function. You order a lumbar spine radiograph and additional diagnostic testing. Which one of the following would be the most helpful next step toward establishing a diagnosis? A. A serum ferritin level and iron studies B. TSH and vitamin B12 levels C. Serum protein electrophoresis D. MRI of the lumbar spine E. A bone marrow biopsy

ANSWER: C This patient's presentation is concerning for hematologic malignancy, in particular multiple myeloma. Along with radiography, the next appropriate step is serum and urine protein electrophoresis. If laboratory studies show a monoclonal spike or if a skeletal survey indicates lytic lesions, a bone marrow biopsy is indicated. MRI of the lumbar spine may also be needed in the future, but it would not discern the etiology of the back pain if lytic lesions were found. Obtaining iron studies, a TSH level, or a vitamin B12 level would not adequately address the initial abnormal laboratory studies or facilitate making the diagnosis of multiple myeloma.

A 50-year-old female sees you for a routine health maintenance visit. On examination you note a thyroid nodule, and evaluation with ultrasonography confirms a solid 1.5-cm nodule. A TSH level is normal. Which one of the following would be the most appropriate next step in management? A. Antithyroid antibody titers B. A radionuclide thyroid uptake scan C. Fine-needle aspiration of the nodule D. Diagnostic surgical excision of the nodule E. Repeat ultrasonography in 6 months

ANSWER: C Thyroid nodules ≥1 cm that are solid or have suspicious features, such as microcalcifications or irregular margins, require fine-needle aspiration (FNA) to rule out malignancy. FNA should not be performed on nodules <1 cm. Antithyroid antibody titers are used in the evaluation of an abnormal TSH level, rather than a thyroid nodule. A low TSH level might also prompt a radionuclide thyroid uptake scan (nuclear medicine thyroid scan) to look for a toxic nodule. These hyperfunctioning nodules are seldom malignant. Surgical nodule excision is not indicated when FNA is negative, but FNA must be performed first to ascertain that. Repeat ultrasonography in 6 months may be warranted after further workup, but FNA is recommended to confirm a diagnosis first.

A 20-year-old football player presents with pain in the proximal fifth metatarsal. The pain was initially present only after practices, but now it causes push-off pain during practice. There is tenderness to palpation. Plain films show no signs of fracture. Which one of the following would be most appropriate at this point? A. Starting NSAIDs and allowing him to continue practicing as tolerated B. Placing him at non-weight bearing status for 2 weeks and repeating the plain films C. Placing him in a hard shoe for 3 weeks and then reevaluating D. Ordering MRI of the foot E. Ordering a bone scan of the foot

ANSWER: D A stress fracture in the proximal fifth metatarsal is particularly prone to nonunion and completion of the fracture. Because complete non-weight bearing or surgical intervention may be necessary with this high-risk fracture, MRI is indicated as the most sensitive test. Bone scans are sensitive but nonspecific. Most stress fractures of the metatarsals occur distally and can be managed with a hard shoe initially, with progressive activity as tolerated. NSAIDs are discouraged because of possible effects on fracture healing.

A 55-year-old male presents to your office to establish care. He has a history of hypertension, hypercholesterolemia, and coronary artery disease. He had a non-ST-elevation myocardial infarction 3 years ago. An echocardiogram at that time was normal and he received a single drug-eluting stent. He has not seen a cardiologist since then and would prefer not to see one unless it is necessary, due to his high insurance copayments for specialist visits. His current medications include clopidogrel (Plavix), aspirin, atorvastatin (Lipitor), lisinopril (Zestril), and carvedilol (Coreg). A review of systems is negative. His vital signs include a blood pressure of 120/72 mm Hg and a heart rate of 80 beats/min. A physical examination is unremarkable, a basic metabolic panel is normal, and his hemoglobin A1c is 5.7%. His LDL-cholesterol level is 60 mg/dL, his HDL-cholesterol level is 49 mg/dL, and his total cholesterol level is 147 mg/dL. The patient would like to reduce the number of medications he takes because of the cost. Which one of the following medications in his current regimen would be most appropriate to discontinue at this time? A. Aspirin B. Atorvastatin C. Carvedilol D. Clopidogrel E.

ANSWER: D After percutaneous coronary intervention (PCI) for a NSTEMI, the American College of Cardiology (ACC) recommends continuing dual antiplatelet therapy (DAPT) for at least 12 months in patients who do not have a high risk for bleeding. There are few consistent recommendations after 12 months because there is less consistent and high-quality evidence for outcomes beyond this time point. The net benefit of DAPT is highest in the first 12 months after PCI. After 12 months of therapy, the risk of bleeding increases. The risk-benefit ratio should be reassessed for all patients after 12 months of therapy. For most patients, discontinuing either the P2Y12 inhibitor or the aspirin is warranted after 12 months. A 2020 meta-analysis found that patients with newer generation drug-eluting stents treated with DAPT for >18 months had a higher all-cause mortality compared with those treated for <6 months. The ACC indicates that continuing DAPT is reasonable for select patients. Point-of-care tools, such as the PRECISE-DAPT Risk Calculator, use clinical information to calculate the likely cardiac risk reduction and likely risk of bleeding to help inform shared decision-making after 12 months. Aspirin is the most cost-effective option for long-term antiplatelet therapy. For this patient, who has indicated costs of care are a concern for him, discontinuing clopidogrel is the better choice. In addition to antiplatelet therapy, control of elevated blood pressure and cholesterol also reduces cardiovascular outcomes in secondary prevention. This patient's LDL-cholesterol level is at goal and currently <70 mg/dL, so atorvastatin should be continued at the current dosage. He has reached his blood pressure goal of <130/80 mm Hg and has no orthostatic symptoms, so his current blood pressure medication regimen should be continued.

An 88-year-old female receiving hospice care is experiencing significant agitation and delirium. After attempting conservative treatments and ruling out reversible causes of her symptoms, which one of the following pharmacologic therapies may be beneficial? A. Alprazolam (Xanax) B. Amitriptyline C. Diphenhydramine (Benadryl) D. Risperidone (Risperdal)

ANSWER: D Agitation and delirium are common end-of-life symptoms. It is important to assess for treatable causes, including constipation, urinary retention, uncontrolled pain, and adverse medication effects. Studies on the treatment of terminal delirium are very difficult to perform and there is no consistently defined pharmacologic treatment. A 2020 Cochrane review called for more research in this area. At this time the antipsychotic medications risperidone and haloperidol have been the most studied and may be effective for treating agitation at the end of life. Benzodiazepines can provoke increased agitation and should be used with caution; however, they can be useful for treating significant end-of-life anxiety. Generally, a longer-acting form such as lorazepam would be a better choice than short-acting alprazolam. Amitriptyline and diphenhydramine can cause other symptoms due to anticholinergic effects.

According to state child abuse mandatory reporter laws, which one of the following is the threshold that must be met for physicians to make a report to local child protective services or law enforcement? A. If they are unable to prove that abuse or neglect did not occur B. If they consider the possibility of abuse or neglect, even if further evaluation indicates that it is unlikely C. If they observe signs or symptoms that may occur in cases of abuse or neglect, but that are nonspecific for abuse or neglect D. If they suspect that a child has experienced abuse or neglect E. Only if they have proof that abuse or neglect has occurred

ANSWER: D Although each state has its own laws regarding obligations to report child abuse, all 50 U.S. states require physicians, whether as a specified professional group or as a part of universal mandated reporting, to report a suspicion of child abuse. The standard is generally suspicion or cause to believe that abuse has occurred. This standard is not met if the possibility of abuse is briefly considered but rejected, or if nonspecific signs are present that do not create a significant suspicion of abuse. There is no burden of proof placed on the physician before making such a report.

A 75-year-old male is noted to have an irregular pulse on examination. He has severe COPD from smoking for many years and uses an albuterol (Proventil, Ventolin) inhaler and tiotropium (Spiriva). On examination his blood pressure is 130/70 mm Hg, and his pulse rate is 110 beats/min and irregularly irregular. Auscultation of the lungs reveals a few scattered wheezes and rhonchi. An EKG shows irregular R-R intervals with narrow QRS complexes and no P waves. A metabolic panel, TSH level, and hemoglobin level are normal, and an echocardiogram shows an ejection fraction of 50%. Which one of the following would be the best initial choice to control this patient's heart rate? A. Cardioversion B. Amiodarone C. Digoxin (Lanoxin) D. Diltiazem (Cardizem) E. Propranolol

ANSWER: D Atrial fibrillation is the most common cardiac arrhythmia and the CDC estimates that more than 12 million persons will have atrial fibrillation by 2030. In patients who are suspected to have atrial fibrillation, a 12-lead EKG should be obtained to confirm the diagnosis. Key treatment decisions include evaluation for possible cardioversion, rate versus rhythm control, and the need for anticoagulation. A rapid rate with atrial fibrillation may cause a reduction in cardiac output by 20%-30%. This is especially problematic in those who already have reduced cardiac output. Patients who need rate control should be treated with a β-blocker such as metoprolol or a nondihydropyridine calcium channel blocker such as diltiazem. Of the choices listed, diltiazem (0.25 mg/kg or 15-20 mg) would provide the best combination of safety and effectiveness for rate control. Cardioversion may be considered if this patient were unstable upon presentation, did not tolerate rate control agents, or had significant symptoms despite achieving rate control. Amiodarone and digoxin could be considered for patients with a low ejection fraction, decompensated heart failure, or mild hypotension. Amiodarone has significant monitoring requirements that would need to be implemented. This patient has COPD, which makes the use of a nonselective β-blocker such as propranolol less desirable.

An older patient presents to your office with findings consistent with polymyalgia rheumatica. This patient is at greatest risk for which one of the following associated conditions? A. Antineutrophil cytoplasmic antibody-associated vasculitis B. Polyarteritis nodosa C. Takayasu arteritis D. Temporal arteritis E. Wegener's granulomatosis

ANSWER: D Both polymyalgia rheumatica (PMR) and giant cell arteritis (also known as temporal arteritis) are chronic inflammatory diseases. PMR is a common chronic inflammatory condition in adults >age 50, although prevalence varies among different countries and populations. Approximately 1 in 5 patients with PMR will have or develop giant cell arteritis, while approximately 50% of patients with giant cell arteritis have or have had PMR. Giant cell arteritis can affect any medium or large artery, particularly the extracranial carotid branches. The temporal artery is often involved, and the ophthalmic artery may also be affected. This can result in neuro-ophthalmic complications, including permanent blindness. For this reason giant cell arteritis with ophthalmic symptoms is considered a medical emergency. Patients with PMR should be evaluated for symptoms of giant cell arteritis, such as visual changes, new-onset headaches, jaw claudication, or scalp tenderness. Giant cell arteritis is confirmed with a temporal artery biopsy, while PMR is diagnosed using a scoring system of signs and symptoms. The treatment of choice for both conditions is high-dose oral prednisone, tapered over months to years as symptoms abate. Antineutrophil cytoplasmic antibody-associated vasculitis, polyarteritis nodosa, Takayasu arteritis, and granulomatosis with polyangiitis (formerly called Wegener's granulomatosis) are not uniquely associated with PMR.

A 33-year-old female presents with palpitations and excessive sweating. A physical examination is normal. Laboratory findings include a TSH (thyrotropin) level of 0.02 µU/mL (N 0.40-4.00) and a free T4 level of 3.9 ng/dL (N 0.7-1.9). Radionuclide scanning reveals no uptake. Which one of the following would explain these findings? A. Thyroid hormone resistance B. Graves disease C. A toxic nodular goiter D. Excess thyroid hormone intake E. A thyrotropin-secreting pituitary tumor

ANSWER: D Excess thyroid hormone intake would cause a low TSH (thyrotropin) level with a high free T4 level and no uptake on radionuclide scan. Other possibilities include an hCG-secreting tumor and the thyrotoxic phase of subacute thyroiditis. An elevated TSH (thyrotropin) level would be seen with thyroid-hormone resistance or a thyrotropin-secreting pituitary tumor. Graves disease causes a homogeneous increased thyroid uptake on radionuclide scanning, whereas a hot nodule would be expected with a toxic nodular goiter.

A 66-year-old male is being discharged after undergoing percutaneous angioplasty and stenting for obstructive coronary artery disease. He is currently asymptomatic. His LDL-cholesterol level is 90 mg/dL. Which one of the following would be most appropriate for secondary prevention of this patient's coronary artery disease? A. No pharmacotherapy B. Evolocumab (Repatha), 140 mg subcutaneously every 2 weeks C. Ezetimibe (Zetia), 10 mg daily D. Rosuvastatin (Crestor), 20 mg daily E. Simvastatin (Zocor), 40 mg daily

ANSWER: D High-intensity statins reduce the risk for recurrent cardiovascular events in patients with established coronary artery disease. Along with lifestyle counseling and cardiac rehabilitation, they should be prescribed to patients who have experienced cardiovascular events. High-intensity statins include atorvastatin, 40-80 mg daily, and rosuvastatin, 20-40 mg daily. Moderate-intensity statins include simvastatin, 20-40 mg daily; atorvastatin, 10-20 mg daily; and rosuvastatin, 5-10 mg daily. Monotherapy with non-statin medications (niacin, ezetimibe, bile acid sequestrants, and fibrates) does not reduce cardiovascular morbidity or mortality. The PCSK9 inhibitors evolocumab and alirocumab are second-line or add-on therapies to maximally tolerated statins. They should be reserved for use in patients who are statin-intolerant or who do not achieve their target LDL-cholesterol level with conventional therapies.

A 13-year-old who was assigned female at birth has been diagnosed with gender dysphoria. His parents fully support their child and affirm his gender as male. On examination the patient has a sexual maturity rating of Tanner stage 3. Which one of the following steps would be appropriate for optimal support and therapy for this patient? A. Encouraging conversion of the patient's gender identity to be congruent with the gender assigned at birth B. Recommending delaying any gender-affirming treatment until he is at least 18 years old to prevent adverse psychosocial outcomes of puberty suppression C. Ordering genetic testing and ultrasonography to confirm the gender assigned at birth D. Recommending GnRH analogue treatment

ANSWER: D Not all family physicians provide gender-affirming care to transgender or gender-diverse adolescents. However, consistent evidence shows adolescents with gender incongruence who receive puberty blockers have improved mental health outcomes compared with adolescents who do not receive this treatment, so it is important for family physicians to recognize the indications for timely treatment or referral. The World Professional Association for Transgender Health (WPATH) recommends medical therapy with GnRH analogues for adolescent patients at Tanner stage 2 or later with significant, sustained gender dysphoria. Patients should have the emotional maturity to be able to understand and consent to therapy, and any comorbid mental health conditions should be controlled enough to allow for informed consent to therapy. Medical therapy for gender dysphoria includes puberty blockers early in puberty, and masculinizing or feminizing therapy, offered later in life. Hormonal intervention therapy should not be offered to transgender children prior to puberty. Puberty suppression is reversible and allows transgender adolescents, their families, and the care team to determine appropriate gender-affirming care as the patient approaches adulthood. It is unethical and against recommended guidelines for a physician to attempt to convert a person's gender identity to the sex assigned at birth. Genetic testing and ultrasonography are not needed to assess this patient.

A 60-year-old former dock worker presents as a new patient to your office for care of his chronic low back pain. He rates his pain as a 3-4 out of 10. He currently takes oxycodone (OxyContin), 10 mg every 12 hours; pregabalin (Lyrica), 150 mg every 12 hours; acetaminophen, 1000 mg every 8 hours; meloxicam (Mobic), 15 mg daily; and cyclobenzaprine, 10 mg every 8 hours. He has been taking these medications for the past 5 years. He tells you that his pain is tolerable but his sex drive and energy level have steadily decreased since starting these medications. Which one of the following medications in this patient's regimen would be most likely to contribute to his decreased libido? A. Acetaminophen B. Cyclobenzaprine C. Meloxicam D. Oxycodone E. Pregabalin

ANSWER: D Opioid use may cause numerous adverse reactions, including drowsiness, pruritus, nausea, and constipation. In addition to these well-known side effects, chronic opioid use can lead to hypogonadism through inhibition of GnRH and an increase in prolactin. A recently published study found that people who use opioids over a long term were nearly twice as likely to be diagnosed with hypogonadism as those who used opioids for shorter, limited timeframes. Given the large number of people who use opioid medications in the United States, prescribers should be aware of this adverse effect and screen for hypogonadism when appropriate. None of the other medications in this patient's regimen, including acetaminophen, cyclobenzaprine, meloxicam, and pregabalin, are the likely cause of his decreased libido.

Pregnancy increases the risk for all of the following dental disorders EXCEPT: A. dental caries B. loose teeth C. gingivitis D. impacted molars E. periodontitis

ANSWER: D Oral health is important to overall health and well-being in all people and has particular importance during pregnancy. The risk of dental caries, loose teeth, gingivitis, and periodontitis all increase during pregnancy. People who are pregnant are at higher risk for dental caries because the oral cavity is exposed to more gastric acidity due to vomiting with morning sickness, and acid reflux due to a lax esophageal sphincter and upward pressure from a gravid uterus. Increased levels of estrogen and progesterone during pregnancy can relax the ligaments and bones that support the teeth, leading to a temporary loosening of teeth. Gingivitis and periodontitis increase during pregnancy because of a fluctuation in estrogen and progesterone levels in combination with changes in oral flora and a decreased immune response. Periodontitis is of particular concern during pregnancy and should be treated because of its association with preterm birth and low birth weight. Impacted molars are a mechanical problem where the tooth does not emerge properly because of abnormal angulation or not enough room in the mouth. Pregnant people may experience impacted third molars (wisdom teeth) because they typically emerge in the late teens or early twenties, but pregnancy itself does not predispose to this condition.

A 23-year-old male presents with a lump in his left testicle that he found while showering last week. He has a history of orchiopexy for cryptorchidism at age 17. He is otherwise healthy and is monogamous with one female sexual partner. On examination you note a 1.5-cm nontender mass in his left testicle. There is no associated swelling of the scrotum and no inguinal lymphadenopathy. Which one of the following would be most appropriate at this time? A. Ciprofloxacin (Cipro), 500 mg twice daily for 14 days B. Serum β-hCG, α-fetoprotein, and LDH levels C. MRI of the scrotum D. Ultrasonography of the scrotum

ANSWER: D Patients with a history of cryptorchidism are at high risk for the development of testicular cancer (relative risk 2.9-6.3), especially if orchiopexy is performed after puberty (odds ratio 5.8). Ultrasonography is the preferred imaging study for the evaluation of a testicular mass. A finding of a solid mass on ultrasonography warrants an urgent urologic referral for the consideration of an orchiectomy. Additional recommended assessments at that time would include measurement of serum tumor markers such as β-hCG, α-fetoprotein, and LDH levels. These are recommended after, not before, ultrasonography. MRI is not typically recommended. If this patient was diagnosed with epididymitis then a fluroquinolone could be considered for treatment. However, this would not be appropriate in this clinical scenario with a nontender mass.

A 22-year-old male reports that he has noticed some small bumps around the tip of his penis and is concerned that he might have a sexually transmitted infection. An examination reveals a row of approximately 2-mm, dome-shaped, skin-colored papules in a ring-like distribution around the corona of the glans penis. Which one of the following is the most likely diagnosis? A. Angiokeratomas B. Genital warts C. Lichen nitidus D. Pearly papules E. Squamous cell carcinoma

ANSWER: D Pearly papules of the penis are a benign, normal anatomic variant and are not sexually transmitted. They are dome-shaped, skin-colored papules 1-4 mm in size with a ring-like distribution around the corona of the glans penis, more commonly found along the dorsal side of the corona. They are present in up to 15%-40% of males. They most commonly occur in late puberty and early adulthood. Circumcised males have a lower incidence. The differential diagnosis includes condyloma acuminata; Tyson glands, which are modified sebaceous glands in a parafrenular distribution; or molluscum contagiosum. Angiokeratomas are well-circumscribed red or blue papules that are 1-6 mm in size. Genital warts are raised masses that can be pearly and smooth or have a rough, cauliflower-like appearance, and are not confined to the penile corona. Lichen nitidus consists of discrete, hypopigmented, 1-mm papules that are not confined to the corona and can also occur on the upper extremities and abdomen. Squamous cell carcinoma may be endophytic (ulcerated) or exophytic (thickened skin or wart-like growths that can progress to a large, irregularly shaped, fungating mass).

A 67-year-old female with hypertension and atrial fibrillation has been taking warfarin for the past 10 years. She has been stable for many years with no complications from her atrial fibrillation or anticoagulation. She is scheduled to undergo elective bladder sling surgery for urinary incontinence. She does not have any other significant past medical history. Which one of the following would be the most appropriate perioperative management of her warfarin? A. Continuing warfarin without interruption B. Discontinuing warfarin 2 days prior to surgery and restarting it 2 days postoperatively unless there is a bleeding complication C. Discontinuing warfarin 2 days prior to surgery and restarting it 5 days postoperatively unless there is a bleeding complication D. Discontinuing warfarin 5 days prior to surgery and restarting it within 24 hours postoperatively unless there is a bleeding complication E. Discontinuing warfarin the day prior to surgery and beginning low molecular weight heparin perioperatively

ANSWER: D Perioperative management of chronic anticoagulation requires an assessment of the patient's risk for thromboembolism and the risk for bleeding from the surgical procedure. Low-risk and typical-risk patients do not require bridging with low molecular weight heparin (LMWH), while it is recommended for high-risk patients. High-risk patients include those with mechanical heart valves and major risk factors for stroke, antiphospholipid antibodies, a stroke or TIA within the previous 3 months, venous thromboembolism within the past 3 months, or coronary artery stent placement within the previous 12 months. High-risk patients whose surgery cannot be delayed require bridging with LMWH. For low-risk patients, warfarin should be discontinued 5 days prior to surgery and restarted within 24 hours postoperatively. This patient's CHA2DS2-VASc score is 3, suggesting that her risk of thromboembolism is relatively low. A patient with atrial fibrillation and a CHA2DS2-VASc score is ≥7 should receive bridging with LMWH.

During an outbreak of head lice at a local school the principal asks you for advice to send home to the parents. In addition to treatment with topical permethrin (Nix), which one of the following would you recommend? A. Using a hairbrush to remove any lice or eggs B. Applying petroleum jelly to the hair and scalp C. Using a conditioner or a combined shampoo and conditioner when applying treatment D. Washing the affected child's recently used clothing and bedding in hot water and drying with hot air E. Treating household pets such as cats and dogs

ANSWER: D Permethrin 1% shampoo is recommended as first-line treatment for head lice. Additional or alternative treatments include topical ivermectin, benzoyl alcohol, and malathion. It is reasonable to wash and heat dry the recently used bedding and clothing of those who have head lice to kill lice and eggs. Head lice can transfer to pillowcases at night and lead to reinfection, but the incidence is very low. The American Academy of Pediatrics recommends that only items that have been in contact with the head of the person with infestation 1-2 days before treatment be considered for cleaning, given the fact that louse survival on inanimate objects beyond 48 hours is extremely unlikely. It is rarely necessary for all other clothing and jackets or coats to be bagged in plastic for 2 weeks as previously recommended. Furniture and carpets may be vacuumed if the infected person used the furniture or was lying on the floor in the previous 2 days. Other recommended measures include removal of any visible nits (eggs) with a nit comb, not a brush. Topical petroleum jelly has only anecdotal evidence of effectiveness. Conditioners can interfere with the action of permethrin, decreasing its effectiveness. Human head lice do not affect pets.

A mother brings in her 4-day-old infant for a well child check. She is primarily breastfeeding him, with occasional formula supplementation. Which one of the following should you advise regarding vitamin D intake for the infant? A. Breastfed infants do not need supplemental vitamin D B. He does not need supplemental vitamin D if he is taking at least 16 oz of formula per day C. Vitamin D supplementation should not be started until he is at least 6 months of age D. He should be given 400 IU of supplemental vitamin D daily E. Intake of vitamin D in excess of 200 IU/day is potentially toxic in infants

ANSWER: D The American Academy of Pediatrics recommends a daily intake of 400 IU of vitamin D in infants, children, and adolescents. Breastfeeding does not provide adequate levels of vitamin D. Exclusive formula feeding provides adequate levels of vitamin D, but infants who consume less than 28 ounces of formula per day need supplementation with 400 IU of vitamin D daily. Vitamin D supplementation can be started once breastfeeding is established, usually within a few days of birth. A growing body of evidence suggests that high-dose maternal vitamin D supplementation (6000-6400 IU daily) is a reasonable alternative to infant supplementation for parents who prefer this approach.

A 13-year-old female is brought to your office with a 3-week history of left groin pain that is most bothersome after she participates in physical education class at her middle school. She does not recall a specific injury and does not participate in extracurricular sports. She had an upper respiratory infection about a month ago but has otherwise been well. An examination reveals a BMI at the 95th percentile for her age. Her vital signs are within normal limits. A musculoskeletal examination is remarkable for limited internal rotation of the hip. Which one of the following is the most likely diagnosis? A. Adductor muscle strain B. Apophysitis of the anterior superior iliac spine C. Legg-Calvé-Perthes disease D. Slipped capital femoral epiphysis E. Transient synovitis

ANSWER: D The most common hip disorder in adolescents (ages 8-15) is slipped capital femoral epiphysis (SCFE). Early diagnosis and treatment are critical in preventing disability related to early-onset degenerative disease of the hip. In the past, SCFE has been more common in boys than in girls, but that prevalence is changing due to the rise in obesity. SCFE should be suspected in an adolescent who has unexplained pain in the hip, groin, thigh, or knee. It is rarely associated with trauma, overuse, or prior illness. On examination the most indicative sign is limited internal rotation of the involved hip. Bilateral radiographs of the hips, including frog-leg lateral views, should be obtained in any adolescent who presents with a new limp and pain in the groin, hip, thigh, or knee (SOR C). Adductor muscle strain (groin strain) is very uncommon in adolescents. Patients suspected of having a groin strain should also undergo radiography. Apophysitis of the anterior superior iliac spine is common in adolescents but is caused by overuse. It is mostly seen in runners, dancers, and ice hockey and soccer players ages 14-18. Legg-Calvé-Perthes disease and transient synovitis are more common in children under age 10. The presenting symptoms of hip pain and a limp are similar to SCFE.

A 16-year-old male sees you for a sports preparticipation examination. His height is 193 cm (76 in) and his weight is 69 kg (152 lb). His measured arm span is 198 cm (78 in). A physical examination reveals a high-arched palate, kyphosis, and pectus excavatum. Which one of the following valvular abnormalities is most likely to develop in this patient? A. Mitral stenosis B. Pulmonic stenosis C. Aortic stenosis D. Aortic insufficiency E. Bicuspid aortic valve

ANSWER: D This adolescent has findings of Marfan syndrome. It is associated with arachnodactyly, an arm span greater than height, a high-arched palate, kyphosis, lenticular dislocation, mitral valve prolapse, myopia, and pectus excavatum. The cardiac examination may reveal an aortic insufficiency murmur or a murmur associated with mitral valve prolapse. Cardiovascular defects are progressive, and aortic root dilation occurs in 80%-100% of affected individuals. Aortic regurgitation becomes more common with increasing age. Mitral stenosis, pulmonic stenosis, aortic stenosis, and bicuspid aortic valve are not associated with Marfan syndrome.

A 40-year-old female presents with a 3-week history of a persistent sore throat despite supportive treatment for a viral upper respiratory infection provided by an urgent care facility. Two COVID-19 polymerase chain reaction tests separated by 5 days were negative during this time. She reports palpitations, weight loss, frequent bowel movements, and anxiety with insomnia for the past month. With further evaluation, you determine that the throat pain she describes is actually localized to the anterior neck when you palpate that area. On examination she has a mildly enlarged thyroid gland. A laboratory evaluation is notable for a suppressed TSH level along with elevated free T4 and total T3 levels. A radioactive iodine uptake scan shows low uptake. Which one of the following is the most likely diagnosis? A. Factitious thyrotoxicosis B. Graves disease C. Multinodular goiter D. Subacute thyroiditis E. TSH-secreting pituitary adenoma

ANSWER: D This patient has findings that are most consistent with a diagnosis of subacute thyroiditis, a self-limited inflammatory condition in which thyroid follicles are destroyed, often in the setting of a recent upper respiratory tract infection. The initial "destructive" phase of subacute thyroiditis presents with signs, symptoms, and laboratory findings of overt hyperthyroidism and is accompanied by a tender thyroid gland and elevation of other inflammatory markers. The radioactive iodine uptake scan is low in these cases. There are emerging case reports in the medical literature of subacute thyroiditis occurring after COVID-19 infection. Graves disease and toxic multinodular goiter also present with overt hyperthyroidism, but radioactive iodine uptake is high in these cases. Factitious thyrotoxicosis is associated with low TSH and elevated or normal free T4 and total T3 levels, but a goiter is not present. A TSH-secreting pituitary adenoma results in elevated TSH, free T4, and total T3 levels. Thyroid receptor antibody (TRAb) assays are very useful for discriminating Graves disease from other causes of thyrotoxicosis. When radiologic studies cannot be done, the total T3 to total T4 ratio and free T3 to free T4 ratio may also help in diagnosis.

A 45-year-old male with a several-month history of fatigue and anorexia with weight loss is found to be anemic, with a hemoglobin level of 9.2 g/dL (N 14.0-18.0). Which one of the following microscopic findings on a peripheral blood smear would be most characteristic of vitamin B12 deficiency anemia in this patient? A. Microcytic, hypochromic RBCs B. Crescent- or sickle-shaped RBCs C. Basophilic stippling of RBCs D. Hypersegmented, polymorphonuclear WBCs E. Schistocytes

ANSWER: D Vitamin B12 deficiency is most commonly related to a deficiency of intrinsic factor that is produced by the gastric mucosa and is necessary for absorption of vitamin B12 in the terminal ileum. Hematologically, it is manifested as a megaloblastic macrocytic anemia. In addition to an elevated mean corpuscular volume, the classic finding on microscopy of the peripheral smear is the presence of multiple (usually five or more) segments in the WBC nuclei. Vitamin B12 levels are generally, but not always, low and a high methylmalonic acid level helps to confirm the diagnosis. Obtaining a methylmalonic acid level is recommended in patients who have normal or low normal vitamin B12 levels but are at risk for deficiency. Risk factors include chronic proton pump inhibitor use; chronic metformin therapy; chronic malnutrition due to alcoholism, chronic gastritis, or peptic ulcer disease; and diseases of the terminal ileum, such as Crohn's disease. People who have had gastric bypass surgery are also at risk for vitamin B12 deficiency, which is why supplementation should be provided in these patients. Microcytic, hypochromic RBCs are typical of iron deficiency anemia but may also be seen in anemia of chronic disease. Crescent- or sickle-shaped cells are seen with sickle cell anemia, and basophilic stippling may be seen in anemia associated with lead toxicity. Schistocytes are seen in hemolytic anemia.

Among patients prescribed metered-dose inhaler treatments, technical errors in using the device are most likely in patients A. 15-30 years of age B. 30-60 years of age C. 60-75 years of age D. ≥75 years of age

ANSWER: D Errors in metered-dose inhaler (MDI) use are a common cause of lack of response to these medications. Studies have documented an increased rate of MDI use errors in certain patient subgroups that could adversely affect the efficacy of treatment. The ability to demonstrate correct MDI use declines significantly with increasing age. One large study of MDI technique, after a minimum of 3 months of prescribed use, found error frequencies of 61% in patients 15-30 years of age, 70% in patients 30-60 years of age, 77% in patients 60-75 years of age, and 86% in patients ≥75 years of age. Other patient characteristics also affect MDI error rates. A higher level of education and a diagnosis of asthma rather than COPD are both associated with fewer errors, whereas the frequency of errors is higher for females and lower-income patients. Disease severity and the presence of comorbidities has also been found to affect MDI use. The error rate is lower for patients who have had prior training, and the rate of proper usage relates directly to the duration of the training session.

A 57-year-old patient who has smoked cigarettes off and on for 30 years is hospitalized and treated with a fluoroquinolone for community-acquired pneumonia. He has no other chronic conditions. Which one of the following could be expected with a 5-day course of antibiotics compared to a longer course in patients such as this? A. Slower clinical improvement B. Higher hospital readmission rates C. Higher mortality rates D. Slower resumption of normal activity E. No difference in clinical outcome

ANSWER: E A 5-day course of antibiotics for community-acquired pneumonia produces the same clinical success rates as longer treatment courses. There is no difference in the rate of clinical improvement, hospital readmissions, or mortality between longer or shorter treatment courses. Patients are often discharged from the hospital before significant clinical improvement occurs, leading both patients and physicians to believe that longer antibiotic courses must be prescribed. Physicians must educate their patients about the benefits of shorter antibiotic courses, including fewer adverse effects, lower cost, and lower rates of bacterial resistance.

Which one of the following is most commonly associated with oligohydramnios? A. Anencephaly B. Esophageal atresia C. Hydrops D. Maternal α-thalassemia E. Posterior urethral valves

ANSWER: E Amniotic fluid volume is regulated in large part by fetal swallowing, inspiration, and urination. Some malformations of the urinary tract, including renal agenesis and persistent obstruction from posterior urethral valves, lead to oliguria or anuria, and are associated with marked oligohydramnios. Anencephaly, esophageal atresia, heart failure, and maternal α-thalassemia are associated with polyhydramnios. Anencephaly is probably the most common cause of polyhydramnios, via transudation from the exposed meninges; swallowing difficulties and excessive urination may also be contributing factors. Esophageal atresia is almost always associated with polyhydramnios due to an inability to swallow. Intrauterine heart failure, whether due to dysrhythmias, structural defects, or severe anemia, often leads to fetal hydrops, which is associated with polyhydramnios. Maternal α-thalassemia can also cause fetal hydrops and polyhydramnios.

You admit an 82-year-old female with an acute ischemic stroke to the hospital. She was not treated with reperfusion therapy with alteplase or thrombectomy because her last known well time is unknown. She has no other acute cardiovascular conditions. You advise the nursing staff to begin treatment with an antihypertensive agent if the patient's blood pressure rises above a threshold of A. 120/80 mm Hg B. 140/90 mm Hg C. 160/100 mm Hg D. 180/110 mm Hg E. 220/120 mm Hg

ANSWER: E Because patients with an acute ischemic stroke may require the increased perfusion pressure to limit ischemia, antihypertensive therapy should not be given during the first 48-72 hours as long as they are not candidates for, or recipients of, reperfusion therapy with alteplase or thrombectomy; do not have a comorbid condition requiring acute blood pressure lowering; and do not have a blood pressure >220/120 mm Hg. Patients with a history of hypertension can generally resume their home blood pressure medications once they are safely eating and drinking.

A 52-year-old pianist is concerned that she may have carpal tunnel syndrome. Which one of the following would be consistent with this problem? A. Weakness of thumb adduction B. Decreased sensation over the thenar eminence C. Decreased sensation over the dorsal aspect of the fourth finger D. Decreased sensation over the dorsal aspect of the fifth finger E. Decreased sensation over the palmar aspect of the thumb, index, and middle finger

ANSWER: E Carpal tunnel syndrome is the most common entrapment neuropathy of the upper extremity. It is caused by compression of the median nerve as it travels through the carpal tunnel. Classically, patients with this condition experience pain and paresthesias in the distribution of the median nerve, which includes the palmar aspect of the thumb, index, and middle fingers, and the radial half of the ring finger. In more severe cases motor fibers are affected, leading to weakness of thumb abduction and opposition. Sensation over the thenar eminence should be normal in patients with carpal tunnel syndrome because it is in the distribution of the palmar cutaneous branch of the median nerve, which branches off proximal to the carpal tunnel.

A 30-year-old female presents for follow-up after an emergency department visit for an episode of symptomatic supraventricular tachycardia that was diagnosed as Wolff-Parkinson-White syndrome. Which one of the following would be most appropriate for the initial long-term management of this patient? A. Adenosine (Adenocard) B. Amiodarone C. Diltiazem (Cardizem) D. Metoprolol E. Referral to a cardiologist for consideration of catheter ablation

ANSWER: E Catheter ablation is the most appropriate treatment for a patient with symptomatic Wolff-Parkinson-White syndrome (WPW). Catheter ablation has a very high immediate success rate (96%-98%). The most significant risk associated with the procedure is permanent atrioventricular block, which occurs in approximately 0.4% of procedures. Adenosine and amiodarone are used for the acute management of supraventricular tachycardia, but not for long-term management. Node-blocking medications such as diltiazem and metoprolol should not be used for the long-term treatment of WPW, due to the increased risk of conduction through the accessory pathway, which may lead to ventricular fibrillation.

You are treating a 57-year-old male who works in a boat manufacturing facility. He has had chronic back pain over the past year, but it has not caused him to miss work. He takes over-the-counter NSAIDs as needed and does not want to take any medications that could reduce his alertness. He asks you what he can to do improve the pain. Which one of the following has been shown to be an effective therapeutic intervention for chronic low back pain? A. Bed rest B. Shoe insoles C. A lumbar brace D. Muscle relaxants E. Physical therapy

ANSWER: E Chronic nonspecific low back pain is a condition with no distinct etiology to explain the patient's associated symptoms. A range of physical activities, including core strengthening, tai chi, and yoga, have been studied for the treatment of nonspecific low back pain. Evidence for most interventions is low quality or inconsistent, although they may be helpful for individual patients and can be offered with shared decision-making to patients without barriers to implementation. Physical therapy has been shown to reduce pain and disability in patients with chronic nonspecific low back pain. Guidelines for low back pain recommend avoiding bed rest. They also recommend avoiding lumbar supports or braces for the long-term treatment or prevention of low back pain due to lack of evidence of benefit. Studies have consistently shown that NSAIDs combined with muscle relaxants have no benefit over NSAIDs alone. Interventions such as shoe insoles have shown little benefit.

Which one of the following must be present for cor pulmonale to develop? A. Obstructive sleep apnea B. COPD C. Right ventricular hypertrophy D. Pulmonary fibrosis E. Pulmonary hypertension

ANSWER: E Cor pulmonale is an alteration in the structure or function of the right heart due to changes in structure or function of the lungs or pulmonary vasculature. Cor pulmonale is the ultimate result of chronic pulmonary hypertension. In its chronic form, longstanding pulmonary hypertension leads to right ventricular hypertrophy. In acute forms, such as pulmonary embolism, acute increases in pulmonary vascular pressures may lead to right ventricular dilation. Virtually any chronic lung or pulmonary vascular condition can cause cor pulmonale. Common parenchymal etiologies include COPD, pulmonary fibrosis, cystic fibrosis, or pneumoconiosis. Vascular etiologies may include primary pulmonary hypertension, recurrent or chronic pulmonary embolism, or collagen vascular disease. Other conditions, such as neuromuscular disease and obesity hypoventilation syndrome, cause chronic hypoxemia, which in turn causes chronic vasoconstriction with resultant pulmonary hypertension. The common denominator, however, is the development of pulmonary hypertension as a consequence of one or more of these conditions, which then leads to cor pulmonale. Most cases are chronic, with resultant right ventricular hypertrophy. The other conditions listed, if untreated over long periods of time, may be associated with pulmonary hypertension but are not necessary for the development of cor pulmonale.

A 68-year-old female comes to your office for a follow-up visit for diabetes mellitus. Her home glucose monitor record shows a range of glucose levels between 68 mg/dL and 125 mg/dL taken twice daily over the last 3 months. Her medications include atorvastatin (Lipitor), 40 mg daily; metformin, 850 mg twice daily; and insulin glargine (Lantus), 10 units nightly. Laboratory studies show a hemoglobin A1c of 5.8% and a creatinine level of 0.98 mg/dL (N 0.6-1.1). She has put much effort into healthy lifestyle behaviors such as walking 30 minutes 5 days per week and avoiding sweetened beverages. Which one of the following would be the most appropriate treatment plan? A. Continuing the current medication regimen B. Increasing the metformin dosage C. Increasing the insulin glargine dosage D. Discontinuing metformin E. Discontinuing insulin glargine

ANSWER: E Insulin glargine increases the risk of hypoglycemia, which is a concern in this patient based on her home glucose monitoring record. Hypoglycemia is associated with an increased risk of mortality in older adults. In addition, insulin is expensive and more complex to administer compared to an oral medication. Since this patient is on a starting dosage of insulin glargine and her hemoglobin A1c is well below her age-adjusted target of 7.5%, she may discontinue insulin glargine and continue metformin as it is well tolerated. The ACCORD (Action to Control Cardiovascular Risk in Diabetes) and ADVANCE (Action in Diabetes and Vascular Disease: Preterax and Diamicron Modified Release Controlled Evaluation) trials showed that aggressive management of diabetes mellitus to achieve a hemoglobin A1c <6.5% increases the risk for patient harm and does not provide clinical benefit. This is not only due to the hypoglycemia itself, but also a result of proinflammatory and prothrombotic compounds induced by the hypoglycemic episodes that are detrimental to vascular health. The ADA recommends a target hemoglobin A1c of 7.0%-7.5% for healthy older adults.

Which one of the following should NOT be consumed during pregnancy due to a potentially high mercury content? A. Catfish B. Crawfish, shrimp, and lobster C. Flounder and haddock D. Salmon and trout E. Shark and swordfish

ANSWER: E Larger ocean fish that consume other fish may accumulate mercury levels that can cause neurologic problems when consumed, so these fish should be avoided by children and people who are pregnant or nursing. Shark and swordfish are among the fish with the highest mercury content. The FDA considers fish and seafood such as catfish, crawfish, shrimp, lobster, flounder, haddock, salmon, and trout to be the best choices of fish for consumption during pregnancy due to their lower mercury content compared to larger ocean fish. Children and patients who are pregnant or nursing are recommended to eat 2-3 servings of low-mercury fish per week.

Lymphadenopathy in which one of the following locations is associated with the highest risk of malignancy? A. Axillary B. Inguinal C. Posterior cervical D. Preauricular E. Supraclavicular

ANSWER: E Lymphadenopathy is a common concern in primary care, and most causes are infectious or inflammatory in nature. Adenopathy that cannot be explained by a concurrent or recent infection or other specific cause is more concerning. Supraclavicular adenopathy is associated with a high risk of intra-abdominal malignancy in both adults and children, with studies finding 34%-50% of these patients having a malignancy. Lymphadenopathy in the other locations listed is associated with a lower risk of malignancy.

A 35-year-old female comes to your office for evaluation of a tremor. During the interview you note jerking movements first in one hand and then the other, but when the patient is distracted the abnormal motion resolves. Aside from the intermittent tremor the neurologic examination is unremarkable. She does not drink caffeinated beverages and takes no medications. Which one of the following is the most likely diagnosis? A. Parkinson disease B. Cerebellar tremor C. Essential tremor D. Physiologic tremor E. Psychogenic tremor

ANSWER: E Psychogenic tremor is characterized by an abrupt onset, spontaneous remission, changing characteristics, and extinction with distraction, such as with alternating finger tapping or mental concentration on serial 7s. Cerebellar tremor is an intention tremor with ipsilateral involvement on the side of the lesion. Neurologic testing will reveal past-pointing on finger-to-nose testing. MRI of the brain is the diagnostic test of choice. Parkinsonian tremor is noted at rest and decreases with voluntary movement. Bradykinesia, rigidity, and postural instability may also be noted. Patients who have essential tremor have symmetric tremors that may involve the hands, wrists, head, voice, or lower extremities. Essential tremor may be treated pharmacologically with propranolol, primidone, or topiramate. Enhanced physiologic tremor is a postural tremor of low amplitude exacerbated by medication. It may also be worse with caffeine use or anxiety.

A 25-year-old landscaper presents to the emergency department with a 1-day history of pain and swelling in his left hand. Three days ago he sustained a puncture wound from a thorn in the palm of his hand. He has generalized achiness and chills in addition to the hand pain. He has no significant past medical history. On examination the patient's temperature is 37.9°C (100.2°F). His left third finger is diffusely swollen, erythematous, and held in flexion. There is tenderness along the third tendon in the palm. A radiograph of the hand is negative for a fracture or foreign body. Which one of the following would be the most appropriate next step? A. Amoxicillin/clavulanate (Augmentin), 875/125 mg twice daily for 10 days B. Incision and drainage in the office with a culture of the wound C. A hand splint and nonurgent referral to an orthopedic surgeon D. Urgent MRI of the hand E. Urgent surgical consultation

ANSWER: E Pyogenic flexor tenosynovitis is an uncommon but serious complication of a penetrating hand injury. The flexor tendon sheath has a poor vascular supply and the synovial fluid is a prime growth medium for bacteria, particularly in patients with a compromised immune system. Flexor tenosynovitis is a clinical diagnosis characterized by the four Kanavel signs: 1) pain with passive extension 2) tenderness with palpation of the tendon sheath 3) flexed position of the involved finger 4) fusiform swelling of the finger. Treatment includes prompt intravenous antibiotics (vancomycin + ceftriaxone) and surgical debridement and irrigation. Without rapid treatment, patients may experience osteomyelitis or disseminated infection requiring amputation. Even with rapid treatment, a substantial percentage of patients have residual loss of range of motion. Flexor tenosynovitis requires urgent surgical consultation and treatment to prevent or reduce the risk of these complications. Patients with suspected flexor tenosynovitis should be seen by a surgeon within 72 hours of symptom onset. Generally, intravenous antibiotics should be started upon diagnosis, but oral antibiotics and splinting of the hand alone are insufficient treatments for the condition. Bedside incision and drainage would also not be sufficient to clear the infection. Delaying surgical consultation to obtain MRI can lead to spreading the infection and permanent loss of function.

A nonverbal 22-year-old male with intellectual disability is brought to your office by the staff of the group home where he lives. They report that the patient has been functioning at his baseline until this morning when he was found to have loud breathing. No other history is available at the time of this visit. On examination he has a temperature of 37.3°C (99.1°F), a blood pressure of 124/82 mm Hg, a pulse rate of 100 beats/min, and a respiratory rate of 16/min. The patient appears to be in mild distress and a high-pitched whistling, crowing sound on inspiration is heard as you walk in the room. Which one of the following would be the most appropriate next step for this patient? A. Oral antibiotics B. Oral corticosteroids C. Nebulized albuterol D. Nebulized epinephrine E. Urgent evaluation in the emergency department

ANSWER: E Stridor is a high-pitched whistling, crowing sound on inspiration. It can be caused by obstruction of the larynx or trachea by a foreign body, vocal cord edema, a neoplasm, or a pharyngeal abscess. Acute stridor requires urgent evaluation to assess for airway obstruction. This patient may have a foreign body or other obstruction in his airway and requires urgent assessment. Oral antibiotics, oral corticosteroids, nebulized albuterol, or nebulized epinephrine would not be appropriate at this time without additional evaluation.

A staff member at a local assisted living facility calls you about an 88-year-old female who has chronic urinary incontinence. A urinalysis and culture were obtained after the patient reported some dizziness and malaise. She does not have dysuria and has had no change to her incontinence. The patient is afebrile and other vital signs are normal. The urine culture reveals >100,000 colony-forming units of Escherichia coli, with sensitivities pending. In addition to supportive care and hydration, which one of the following would be indicated at this time? A. Ciprofloxacin (Cipro) B. Fosfomycin (Monurol) C. Nitrofurantoin (Macrodantin) D. Sulfamethoxazole/trimethoprim (Bactrim) E. No antibiotics

ANSWER: E This patient has asymptomatic bacteriuria (ASB) and does not require antibiotic therapy at this time. In women age >70 the incidence of ASB is nearly 20%, and in patients with chronic incontinence and older adults with disabilities rates may be as high as 50%. Symptoms consistent with a urinary tract infection (UTI) include acute dysuria, new or worsening urinary urgency or frequency, new incontinence, gross hematuria, and suprapubic or costovertebral angle tenderness. The Infectious Diseases Society of America recommends close observation without antibiotic treatment for patients with bacteriuria who have non-urinary symptoms such as delirium, malaise, or fall as long as they do not have signs of serious systemic infection such as fever or sepsis. It is reasonable to hold off on ordering a urinalysis in a patient without UTI-specific symptoms who also does not have serious systemic signs. When antibiotic therapy is indicated for a true UTI, empiric therapy is often guided by recommendations based on local antibiotic resistance patterns, which can vary widely. Generally, agents with Escherichia coli resistance rates >10%-20% should not be used as empiric therapy. Nitrofurantoin is a common recommendation but should be used with caution in patients with a creatinine clearance of 30-60. TMP-SMX is also recommended unless local resistance patterns indicate otherwise. Historically fluoroquinolones have also been effective, but resistance rates have been rising and thus may not be recommended in some communities. Fosfomycin and first-generation cephalosporins are also first-line empiric therapy options without widespread resistance, although fosfomycin can be cost-prohibitive. Once a culture and sensitivity result is known, antibiotic choice can be tailored to the results.

A 56-year-old male comes to your office because of right shoulder pain for the past week. It started when he lifted a heavy piece of furniture while helping a friend move. He felt immediate sharp pain in his shoulder, which has since radiated down the biceps toward the right radial forearm. On examination there is no deformity of the shoulder or arm. He has increased pain with palpation in the anterior shoulder near the bicipital groove of the humerus. You suspect biceps tendinitis. Anterior shoulder pain with which one of the following examination maneuvers of the right arm would be most consistent with this diagnosis? A. Active or passive cross adduction of the arm at the shoulder B. Shooting pain to the thumb with axial compression of the head with the neck in extension and rotated or flexed toward the right shoulder C. Resisted extension of the elbow with the shoulder in a neutral position D. Resisted internal rotation of the shoulder with the elbow flexed to 90° E. Resisted supination of the hand with the elbow flexed to 90°

ANSWER: E This patient's symptoms are consistent with biceps tendinitis, which causes pain with abduction and external rotation of the arm, and tenderness of the bicipital groove with palpation. Resisted supination of the hand with the elbow flexed to 90° is the Yergason test, and anterior shoulder pain with this maneuver is consistent with bicipital tendinitis. Anterior shoulder pain with cross adduction of the arm is more consistent with acromioclavicular arthritis. Axial compression with rotation to the affected side of the slightly extended neck is the Spurling test for cervical radiculopathy. Extension of the elbow would activate the triceps, and internal rotation of the shoulder with the elbow flexed would result in less activation of the biceps than resisted supination.

Which one of the following treatments has been shown to improve the quality of life for a patient with tinnitus? A. Antidepressant therapy B. Ginkgo biloba C. Niacin D. Vitamin B12 E. Cognitive behavioral therapy

ANSWER: E Treatments to reduce awareness of tinnitus and tinnitus-related distress include cognitive behavioral therapy, acoustic stimulation, and educational counseling. No medications, supplements, or herbal remedies have been shown to substantially reduce the severity of tinnitus.

A 4-month-old female is brought to your office by her mother for a well child visit. The mother tells you about some red patches on the child's cheeks and legs that do not seem to bother the infant. She says that the patches sometimes appear very irritated and improve with occasional lotion use but keep coming back. The mother has not noticed any signs of illness. An examination reveals a well appearing infant with normal growth, development, and vital signs. You note slightly rough, erythematous patches on both cheeks and her chin, as well as on her thighs. Which one of the following would be most appropriate at this time? A. Twice-daily application of a fragrance-free moisturizer with a high lipid-to-water ratio B. Twice-daily application of a low-potency topical corticosteroid C. Application of a topical calcineurin inhibitor for the facial lesions and a low-potency topical corticosteroid for the other areas D. Allergy testing E. Referral to a dermatologist

ANSWER: A This infant has skin findings that are consistent with atopic dermatitis, a common skin condition that typically presents in the first year of life with erythematous patches or plaques. In young children, the distribution of the rash is most commonly seen on the face, scalp, trunk, and extremities. In older children, the flexural surfaces are more affected. The first-line treatment is liberal use of fragrance-free emollients, at least 1-2 times per day. Emollients with a high lipid-to-water ratio are the most effective; ointments have the highest ratios, followed by creams and then lotions. A low-potency topical corticosteroid is an appropriate treatment for more significant flares that are not controlled by emollients. Topical calcineurin inhibitors are not approved for use in children <2 years of age. Allergy testing is not recommended for the routine evaluation of atopic dermatitis. A specialty referral is not necessary for straightforward atopic dermatitis but is recommended for patients with a poor response to appropriate first-line treatment, severe or recurrent skin infections, significant psychosocial problems due to atopic dermatitis, an uncertain diagnosis, or uncontrolled facial atopic dermatitis.

The U.S. Preventive Services Task Force recommends vision screening to detect amblyopia A. at 2 years of age B. at 3-5 years of age C. the summer before the child enters first grade D. once at 4-5 years of age and once at 8-9 years of age

ANSWER: B Amblyopia is one of the most common causes of vision abnormalities in children, and early detection and treatment can help prevent vision loss. The U.S. Preventive Services Task Force recommends vision screening for all children at least once between 3 and 5 years of age to detect the presence of amblyopia or its risk factors (B recommendation).

A 12-year-old female is brought to your office as a new patient for evaluation of an earache. During the evaluation you learn that the child has been living in a household where there was interpersonal violence impacting her mother. You are told that the child's father is now incarcerated. Considering how adverse childhood experiences affect behavior and health, this child is at greatest risk for which one of the following? A. Alcohol use disorder B. Attention-deficit disorder C. Borderline personality disorder D. Dissociative disorder E. Schizophrenia

ANSWER: A Of the options listed, this child is at greatest risk for alcohol use disorder as a consequence of adverse childhood experiences (ACEs). Many health-related factors are associated with cumulative ACEs. As ACEs increase so do the risks for alcohol use disorder, substance use disorder, depression, suicide attempts, smoking, poor self-rated health, and sexually transmitted infections. ACEs are also associated with multiple medical conditions that can contribute to chronic disease burden and even premature mortality. These can include diabetes, cancer, heart disease, and chronic lung disease.

During a newborn examination you note a foot deformity, with the front half of the foot turned inward. Applying gentle pressure to the forefoot while holding the heel steady brings the heel and forefoot into alignment. Which one of the following would you recommend? A. Observation only B. Adjustable shoes C. Serial casting D. Surgical correction

ANSWER: A This patient has flexible metatarsus adductus, the most common congenital foot deformity. Flexible metatarsus adductus usually resolves spontaneously by 1 year of age and does not require treatment. Rigid metatarsus adductus should be treated with serial casting. Using adjustable shoes is an alternative that is less expensive than serial casting for motivated parents with children who are not yet walking. Surgical correction should be reserved for older children who are already walking or for those with persistent symptomatic metatarsus adductus that is resistant to casting.

A 75-year-old patient is admitted to the hospital. The Joint Commission National Patient Safety Goals program requires medication reconciliation for this patient both on admission and at the time of discharge. The primary intent of this reconciliation is to detect: A. potentially inappropriate medication use in the elderly B. high-risk medication use C. medication discrepancies D. polypharmacy E. adverse medication effects

ANSWER: C Maintaining and communicating accurate patient medication information is one of the goals of the Joint Commission National Patient Safety Goals program. This includes medication reconciliation, which is intended to identify and resolve discrepancies. In this process, a clinician compares the medications a patient should be using and is actually using with the new medications that are ordered. While potentially inappropriate medication use in the elderly, high-risk medication use, polypharmacy, or adverse medication effects might also occur and might be beneficial to address, these are not the primary focus of medication reconciliation.

A 58-year-old male with a history of tobacco use disorder and alcohol use disorder presents with the sudden onset of many well circumscribed brown, oval, rough papules with a "stuck-on" appearance on his trunk and proximal extremities (see image). On examination you also note an unintentional 6-kg (13-lb) weight loss over the last 3 months and conjunctival pallor. A review of systems is positive for abdominal pain, decreased appetite, and mild fatigue. You order a laboratory workup. Which one of the following would be most appropriate at this point?

A. Reassurance that the skin lesions are benign B. A skin biopsy C. Referral to a dermatologist D. CT of the abdomen and pelvis E. Upper and lower endoscopy . . . . . . . . . . . . . . . . . . . . . . . . . . . . . ANSWER: E The Leser-Trélat sign may be defined as the abrupt onset of multiple seborrheic keratoses, which is an unusual finding that often indicates an underlying malignancy, most commonly an adenocarcinoma of the stomach. This patient's age, risk factors, red-flag symptoms, and other clinical findings indicate the need for endoscopy. Further skin evaluation and lifestyle changes, which are indicated, will not address the need for evaluation of weight loss and other abnormal symptoms and findings. CT is not an initial approach for diagnosing a suspected malignancy of the stomach or colon.

A 60-year-old patient is admitted to the hospital for a COPD exacerbation. COVID-19 testing is negative. For this patient, which one of the following would be the most appropriate prednisone dosage? A. 40 mg daily for 5 days B. 40 mg daily for 10 days C. 60 mg daily, tapered over 6 days D. 60 mg daily for 10 days

ANSWER : A For patients with a COPD exacerbation, systemic glucocorticoids can improve FEV1, increase oxygenation, shorten recovery time, and reduce the length of hospitalization (level of evidence A). Prednisone, 40 mg daily for 5 days, is recommended for COPD exacerbations. Longer courses are associated with an increased risk of pneumonia and mortality. Studies have shown that oral administration is equally efficacious compared to the intravenous route. The 2023 GOLD recommendations also state that antibiotics for 5-7 days should be given if there is either an increase in dyspnea or sputum volume along with an increase in sputum purulence.

A 45-year-old female presents with left shoulder pain. On examination she has pain and relative weakness when pushing toward the midline against resistance while the shoulder is adducted and the elbow is bent to 90°. With the elbow still at 90° she is unable to keep her left hand away from her body when you position her hand behind her back. This presentation is most consistent with an injury of which one of the following tendons? A. Deltoid B. Infraspinatus C. Subscapularis D. Supraspinatus E. Teres minor

ANSWER : C This patient's pain and weakness while pushing against resistance reveals weakness on internal rotation of the shoulder, which suggests a possible tear of the subscapularis tendon. The inability to keep her hand away from her body when it is placed behind her back describes a positive internal lag test, also suggesting involvement of the subscapularis tendon. The infraspinatus and teres minor are involved in external rotation rather than internal rotation. The deltoid and supraspinatus are involved in abduction of the shoulder.

A 50-year-old female sees you for follow-up of her hypertension. At her last visit 4 weeks ago you started her on lisinopril (Zestril), 10 mg daily, because of a blood pressure of 158/92 mm Hg that was confirmed with home blood pressure monitoring. She is tolerating the medication well and has no side effects. She does not take any other medications and has no other chronic medical conditions. Today her blood pressure is 149/90 mm Hg, which you confirm on repeat measurement. This is also consistent with her home measurements. At her last visit a basic metabolic panel was normal. You repeat a basic metabolic panel today and the results are normal except for a BUN level of 25 mg/dL (N 8-23) and a creatinine level of 1.5 mg/dL (N 0.6-1.1). At her last visit her BUN level was 12 mg/dL and her creatinine level was 0.7 mg/dL. Which one of the following would be most appropriate at this time? A. Continuing her current treatment regimen B. Increasing lisinopril to 20 mg daily C. Continuing lisinopril at the current dosage and adding amlodipine (Norvasc), 5 mg daily D. Discontinuing lisinopril and beginning amlodipine, 5 mg daily E. Discontinuing lisinopril and beginning losartan (Cozaar), 25 mg daily

ANSWER : D This patient has more than a 30% rise in her creatinine level and has no other indication for ACE inhibitor or angiotensin receptor blocker (ARB) therapy for her hypertension. Discontinuing lisinopril and starting a medication from a different class is therefore the most appropriate treatment at this time. The clinical finding of an abruptly rising creatinine level >30% from baseline after the initiation of an ACE inhibitor or ARB may be associated with underlying renal disease such as renal artery stenosis. The rise in creatinine level is associated with an increased risk of renal failure, adverse cardiac outcomes, and death, and the medication should be stopped pending evaluation. A 2017 study suggests that rises in serum creatinine of <30% also put patients at risk for these outcomes, with a dose-response relationship between the magnitude of creatinine change and the risk of adverse outcomes. Renin-angiotensin system blockade has also been associated with long-term benefits, and balancing the benefits and risks of these medications may be complex. Patients with other indications for ACE inhibitors and ARBs such as heart failure benefit from individualized decision-making in this regard. Appropriate alternatives include a calcium channel blocker such as amlodipine or a thiazide-type diuretic.

A 34-year-old female has posttraumatic stress disorder that started 6 months ago after she was the victim of an armed robbery. Her symptoms include nightmares, anxiety, hypervigilance, and some paranoia, and she has been reexperiencing the robbery. She has been receiving psychotherapy but would now like to start a medication. Which one of the following would be the most appropriate medication to start? A. Bupropion (Wellbutrin) B. Buspirone C. Lorazepam (Ativan) D. Risperidone (Risperdal) E. Venlafaxine (Effexor XR)

ANSWER : E Psychotherapy reduces arousal, hypervigilance, sleep disturbances, and depression symptoms in patients with posttraumatic stress disorder (PTSD). For those who continue to have PTSD symptoms, SSRIs and venlafaxine are considered first-line medications. Bupropion and buspirone have not been found to be effective treatments. The use of benzodiazepine medications is not recommended because of the high risk of misuse. Antipsychotic medications such as risperidone would not be appropriate as first-line pharmacotherapy.

Which one of the following is the fundamental counseling strategy used in the Stages of Change Model? A. Quickly establishing rapport with a patient to improve compliance with recommendations for change B. Assessing the patient's motivation for change and determining where they are in the process C. Focusing on a specific aspect of a problem and offering strategies for coping D. Providing education regarding the behavior in which a change is recommended E. Providing direct advice regarding steps for making a lifestyle change

ANSWER: B The Stages of Change Model assesses the patient's motivation for change and determines which stage of the change process the patient is in. The stages include precontemplation, contemplation, preparation, action, and maintenance. Understanding this helps guide various counseling strategies that family physicians can utilize for each individual patient.

Of the following dietary herbal supplements, which one carries the highest risk for clinically important medication interactions? A. Black cohosh B. Ginseng C. St. John's wort (Hypericum perforatum) D. Saw palmetto E. Valerian

ANSWER: C St. John's wort can reduce the effectiveness of multiple medications because it is a potent inducer of CYP3A4 and P-glycoprotein synthesis. Concurrent use of St. John's wort with medications that are metabolized by these systems should be avoided. These include cyclosporine, warfarin, theophylline, and oral contraceptives. Black cohosh, ginseng, saw palmetto, and valerian do not have CYP3A4 activity.

A 10-year-old male is brought to the emergency department with a history of group A β-hemolytic Streptococcus confirmed with a throat culture 2 weeks ago. His parents gave him antibiotics for 3 days then stopped them because his symptoms were gone. He now has a temperature of 38.9°C (102.0°F), a heart rate of 122 beats/min, and right hip and left knee pain with swelling. A targetoid erythematous rash is present across his chest and back. In addition to resuming antibiotic therapy, which one of the following would be the most appropriate initial pharmacologic therapy for this condition in this patient? A. Acetaminophen B. Gabapentin (Neurontin) C. Hydrocodone D. Naproxen

ANSWER: D Fever, erythema marginatum, and polyarthritis within 7-14 days of acute strep pharyngitis is consistent with acute rheumatic fever. Once the diagnosis of acute rheumatic fever is made, NSAIDs such as naproxen (10 to 20 mg/kg/day in two divided doses) should be administered along with appropriate antibiotic therapy. The therapeutic response of arthritis to NSAIDs is often remarkable. Acetaminophen has not been shown to be a superior analgesic for acute rheumatic fever. Gabapentin is not indicated, especially considering that the pain does not have a neuropathic etiology. Opioids would not be considered first-line treatment because of their adverse effects and the usual dramatic response to NSAIDs alone.

A 40-year-old female presents for follow-up of severe asthma. Skin testing for common inhalant allergens is negative, and her serum IgE levels are normal. She has a history of rhinorrhea and wheezing with exposure to aspirin. Which one of the following additional conditions is most likely to be present? A. Allergic rhinitis B. Aspergillosis C. Eczema D. Nasal polyps E. Obesity

ANSWER: D This patient has aspirin/NSAID drug-exacerbated respiratory disease (AERD), a chronic respiratory condition with three clinical features: asthma, sensitivity to aspirin and NSAIDs, and sinus disease with recurrent nasal polyps. It tends to have an acute onset in adulthood, most commonly ages 20-50, and the asthma is often severe and persistent. This type of asthma may also be referred to as nonallergic asthma or intrinsic asthma and affects approximately 10% of patients with asthma. Patients with intrinsic asthma commonly also have nasal polyps. Allergic rhinitis and eczema are both associated with atopy and patients with these conditions would also have positive skin tests for inhalant allergens and elevated IgE levels. While aspergillosis is associated with chronic sinusitis and nasal polyps, skin tests and IgE levels would also be positive. Obesity is an independent risk factor for asthma but is not specifically related to intrinsic asthma.

When performing a geriatric assessment, which one of the following is an instrumental activity of daily living? A. Bathing B. Dressing C. Transferring between the bed and a chair D. Using the telephone E. Using the toilet

ANSWER: D The foundation of geriatric assessment is evaluating the individual's ability to perform tasks required for living. Instrumental activities of daily living include activities necessary to live independently, such as using a telephone, doing housework, preparing meals, taking medications properly, and managing finances. Activities of daily living are self-care activities that are performed daily, such as eating, bathing, dressing, transferring between the bed and a chair, and toileting, including bladder and bowel function.

A 34-year-old female comes to your office for follow-up after an emergency department visit because of anxiety. She notes persistent anxiety, poor focus, and palpitations. She also reports that she is not hungry and has lost several pounds. She reports "odd things happening" such as sudden weakness in her legs, falling, and getting lost. When taking her history you note that the patient is hyperverbal and displays tangential speech. She has a temperature of 37.4°C (99.3°F), a heart rate of 134 beats/min, a respiratory rate of 20/min, and a blood pressure of 117/69 mm Hg. A physical examination reveals an anxious-appearing female who is tremulous at rest. An HEENT examination shows exophthalmos and no thyromegaly. A cardiac examination is unremarkable aside from tachycardia. A pulmonary examination reveals faint bibasilar crackles. An EKG shows sinus tachycardia. Laboratory results are as follows: CBC............within normal limits Basic metabolic panel............within normal limits TSH............<0.08 µU/mL (N 0.35-3.00) Free T4............4.51 ng/dL (N 0.89-1.80) Free T3............>19.0 pg/dL (N 2.3-4.2) Which one of the following would be the most appropriate initial step? A.

ANSWER: E This patient is experiencing tachycardia, confusion, and pulmonary edema in the setting of hyperthyroidism. This is consistent with findings of thyrotoxicosis, also known as thyroid storm. This rare, life-threatening endocrine condition requires emergent management in a hospital setting. Other symptoms of thyroid storm include fever, central nervous system dysfunction, gastrointestinal or liver dysfunction, and cardiovascular complications such as heart failure. The Burch-Wartofsky Point Scale is a diagnostic support tool that assigns a point value to symptoms based on their presence and severity. This patient's most pressing need is medical stabilization including supportive care and β-blockade. It would be inappropriate to start treatment with an anti-thyroid agent such as methimazole prior to hospitalization. While a thyroid receptor antibody test may be useful in identifying the cause of the condition it should not delay hospitalization. A radioactive iodine uptake test is also useful for identifying the underlying cause of hyperthyroidism but should be avoided until the thyroid storm has resolved. Consultation with an endocrinologist is important but should occur in the inpatient setting.

A 43-year-old female presents to your office with a 3-month history of left low back and posterior hip pain. She does not recall an injury but says she was very active during a move to a new home prior to the onset of the pain. An examination reveals that her gait, lower extremity strength, and hip and knee range of motion are normal. A straight leg raising test is negative, and a log roll test is also normal. A flexion, abduction, and external rotation (FABER) test produces pain in the low back area. Which one of the following is the most likely diagnosis? A) Femoroacetabular impingement B) Greater trochanteric pain syndrome C) Osteoarthritis D) Piriformis syndrome E) Sacroiliac joint dysfunction

ANSWER: E The cause of hip pain is generally determined from the patient's history and a physical examination. A positive flexion, abduction, and external rotation (FABER) test that produces pain at the sacroiliac joint, lumbar spine, and posterior hip is associated with sacroiliac joint dysfunction. The log roll test involves passive supine internal and external rotation of the hip. When this test is positive for pain it is associated with hip joint disease. While femoroacetabular impingement may be associated with a positive FABER test, it would produce pain in the groin, not the low back. Greater trochanteric pain syndrome results in lateral hip pain rather than posterior hip pain. Osteoarthritis is usually associated with a limited range of motion and groin pain. Piriformis syndrome is evaluated with the seated piriformis stretch test.

A 58-year-old female presents to your office after being seen in the emergency department last weekend for her first episode of renal colic. She later passed a calcium phosphate kidney stone. Which one of the following medications in her current regimen places her at higher risk for kidney stone formation? A. Escitalopram (Lexapro) B. Levothyroxine (Synthroid) C. Lisinopril (Zestril) D. Metformin E. Topiramate (Topamax)

ANSWER: E Topiramate increases the risk of kidney stones. It is a carbonic anhydrase inhibitor, which induces a metabolic acidosis that leads to hypercalciuria and increases the risk of calcium phosphate stones. Patients who are taking topiramate should avoid dehydration and ketogenic diets to reduce the risk of stone formation. The risk of kidney stones is not increased by escitalopram, levothyroxine, lisinopril, or metformin.

A 5-year-old male is brought to your office after passing an intestinal worm. He lives on a farm with cattle, pigs, and dogs. He has never traveled very far from home. He does not have any respiratory symptoms or diarrhea, but has experienced some abdominal bloating. His parents bring a picture of the worm (shown below). Which one of the following is the infecting organism?

A. Ascaris lumbricoides (roundworm) B. Enterobius vermicularis (pinworm) C. Giardia lamblia D. Necator americanus (hookworm) E. Taenia solium (tapeworm) . . . . . . . . . . . . . . . . . . . . . . . . . . . . . ANSWER: A This case and image are consistent with Ascaris lumbricoides infestation. A. lumbricoides is a large roundworm that typically infects the ileum. Symptoms are variable but large infections can lead to intestinal obstruction. Pinworms (Enterobius vermicularis) are much smaller and typically present with anal pruritus. Giardia lamblia is a microscopic protozoan parasite that is not visible on gross examination. Hookworms (Necator americanus) are also round, but are typically 6-12 mm in length. They are a significant cause of anemia in children globally. Tapeworms can be large, but are flat and segmental in appearance, and are typically found in the stool as segments called proglottids.

You are treating a 64-year-old female for pyelonephritis. She has previously been in good health with no chronic health issues. The patient appears acutely ill but is answering questions appropriately. On examination her weight is 100 kg (220 lb), her temperature is 38.9°C (102.0°F), her pulse rate is 110 beats/min, her respiratory rate is 24/min, her blood pressure is 136/72 mm Hg, and her oxygen saturation is 94% on room air. Initial laboratory findings include an elevated WBC count of 14,550/mm3 (N 4000-11,000) and a venous lactate level of 4.3 mmol/L (N 0.6-1.7). You decide to start lactated Ringer's solution intravenously. Which one of the following would be the most appropriate initial fluid resuscitation rate? A. 100 mL/hr B. 150 mL/hr C. 200 mL/hr D. 3000 mL over 30 minutes E. 3000 mL over 3 hours

ANSWER: E The Surviving Sepsis Campaign recommends that patients with elevated serum lactate or hypotension receive crystalloid fluids such as lactated Ringer's solution at an initial rate of 30 mL/kg in the first 3 hours using boluses of approximately 500 mL. A serum lactate value >4 mmol/L (36 g/dL) is correlated with increased severity of illness and poorer outcomes even if hypotension is not yet present. Patients who are hypotensive or whose serum lactate level is >4 mmol/L require intravenous fluids or colloid to expand their circulating volume and effectively restore perfusion pressure. The administration of 30 mL/kg of fluid is recommended as a fluid challenge, which should be started as early as possible in the course of septic shock. The choice of crystalloid fluid is evolving. Historically normal saline was preferred, but recent studies have suggested that balanced crystalloids, such as lactated Ringer's solution, improve outcomes including mortality when compared with normal saline.

A 34-year-old female comes to your office for follow-up after an emergency department visit because of anxiety. She notes persistent anxiety, poor focus, and palpitations. She also reports that she is not hungry and has lost several pounds. She reports "odd things happening" such as sudden weakness in her legs, falling, and getting lost. When taking her history you note that the patient is hyperverbal and displays tangential speech. She has a temperature of 37.4°C (99.3°F), a heart rate of 134 beats/min, a respiratory rate of 20/min, and a blood pressure of 117/69 mm Hg. A physical examination reveals an anxious-appearing female who is tremulous at rest. An HEENT examination shows exophthalmos and no thyromegaly. A cardiac examination is unremarkable aside from tachycardia. A pulmonary examination reveals faint bibasilar crackles. An EKG shows sinus tachycardia. Laboratory results are as follows: CBC............w

ANSWER: E This patient is experiencing tachycardia, confusion, and pulmonary edema in the setting of hyperthyroidism. This is consistent with findings of thyrotoxicosis, also known as thyroid storm. This rare, life-threatening endocrine condition requires emergent management in a hospital setting. Other symptoms of thyroid storm include fever, central nervous system dysfunction, gastrointestinal or liver dysfunction, and cardiovascular complications such as heart failure. The Burch-Wartofsky Point Scale is a diagnostic support tool that assigns a point value to symptoms based on their presence and severity. This patient's most pressing need is medical stabilization including supportive care and β-blockade. It would be inappropriate to start treatment with an anti-thyroid agent such as methimazole prior to hospitalization. While a thyroid receptor antibody test may be useful in identifying the cause of the condition it should not delay hospitalization. A radioactive iodine uptake test is also useful for identifying the underlying cause of hyperthyroidism but should be avoided until the thyroid storm has resolved. Consultation with an endocrinologist is important but should occur in the inpatient setting.

The U.S. Preventive Services Task Force recommends one-time screening for abdominal aortic aneurysm in:

The U.S. Preventive Services Task Force (USPSTF) concluded with moderate certainty that there is a net benefit for screening for abdominal aortic aneurysm (AAA) in 65- to 75-year-old men who have ever smoked (defined as >100 lifetime cigarettes) (B recommendation). For men in this same age group who have never smoked, the USPSTF recommends that clinicians selectively offer screening for AAA after consideration of other risk factors (C recommendation), but they do not address the factors that affect screening recommendations for this group. The USPSTF concluded at this time that there is insufficient evidence (I statement) that screening women who have ever smoked is beneficial. These statements reflect a lack of good studies to assess overall benefit or harm in specific populations. The USPSTF recommends against screening (D recommendation) in women without a smoking history because the harms outweigh the benefits. The primary method of screening for AAA is conventional abdominal duplex ultrasonography.


Set pelajaran terkait

Final (biod 121 Portage Nutrition)

View Set

Spanish FORMAL COMMANDS (Mandatos formales / usted - ustedes) (Telling someone you refer to formally or a group of people what to do or not do) (changing the infinitive to the formal command)

View Set

Chap 20 : Assessment of Respiratory Function

View Set

MicroEcon 247 Practice Final Exam Part 1 (all MindTap Quizlets into one)

View Set

بنية النص الحجاجي

View Set

Saunders Comprehensive Review for NCLEX-PN - Chapter 6

View Set

Ch. 13 Learnsmart Consumer Behavior

View Set